ChaseDream

标题: 【每日逻辑练习第二季】【1-13】 [打印本页]

作者: bananazoo    时间: 2011-12-2 00:27
标题: 【每日逻辑练习第二季】【1-13】
恩,发作业啦~

【精练】
2. High school students who feel that they are not
succeeding in high school often drop out before
graduating and go to work. Last year, however, the
city’s high school dropout rate was significantly
lower than the previous year’s rate. This is
encouraging evidence that the program instituted
two years ago to improve the morale of high school
students has begun to take effect to reduce dropouts.
Which one of the following, if true about the last
year, most seriously weakens the argument?
----------Cause and Effect Problems NO.2

(A) There was a recession that caused a high level
of unemployment in the city.
(B) The morale of students who dropped out of
high school had been low even before they
reached high school.
(C) As in the preceding year, more high school
students remained in school than dropped
out.
(D) High schools in the city established placement
offices to assist their graduates in obtaining
employment.
(E) The antidropout program was primarily aimed
at improving students’ morale in those high
school with the highest dropout rates.

【逻辑链】
49.

An overly centralized economy, not the changes in the climate, is responsible for the poor agricultural production in Country X since its new government came to power.  Neighboring Country Y has experienced the same climatic conditions, but while agricultural production has been falling in Country X, it has been rising in Country Y.
Which of the following, if true, would most weaken the argument above?
(A) Industrial production also is declining in Country X.
(B) Whereas Country Y is landlocked, Country X has a major seaport.
(C) Both Country X and Country Y have been experiencing drought conditions.
(D) The crops that have always been grown in Country X are different from those that have always been grown in Country Y.
(E) Country X's new government instituted a centralized economy with the intention of ensuring an equitable distribution of goods.

50.
Generally scientists enter their field with the goal of doing important new research and accept as their colleagues those with similar motivation.  Therefore, when any scientist wins renown as an expounder of science to general audiences, most other scientists conclude that this popularizer should no longer be regarded as a true colleague.
The explanation offered above for the low esteem in which scientific popularizers are held by research scientists assumes that
(A) serious scientific research is not a solitary activity, but relies on active cooperation among a group of colleagues
(B) research scientists tend not to regard as colleagues those scientists whose renown they envy
(C) a scientist can become a famous popularizer without having completed any important research
(D) research scientists believe that those who are well known as popularizers of science are not motivated to do important new research
(E) no important new research can be accessible to or accurately assessed by those who are not themselves scientists

51.
A company's two divisions performed with remarkable consistency over the past three years:  in each of those years, the pharmaceuticals division has accounted for roughly 20 percent of dollar sales and 40 percent of profits, and the chemicals division for the balance.

Which of the following can properly be inferred regarding the past three years from the statement above?
(A) Total dollar sales for each of the company's divisions have remained roughly constant.
(B) The pharmaceuticals division has faced stiffer competition in its markets than has the chemicals division.
(C) The chemicals division has realized lower profits per dollar of sales than has the pharmaceuticals division.
(D) The product mix offered by each of the company's divisions has remained unchanged.
(E) Highly profitable products accounted for a higher percentage of the chemicals division's sales than of those of the pharmaceuticals division.

52.
The technological conservatism of bicycle manufacturers is a reflection of the kinds of demand they are trying to meet.  The only cyclists seriously interested in innovation and willing to pay for it are bicycle racers.  Therefore, innovation in bicycle technology is limited by what authorities will accept as standard for purposes of competition in bicycle races.
Which of the following is an assumption made in drawing the conclusion above?
(A) The market for cheap, traditional bicycles cannot expand unless the market for high-performance competition bicycles expands.
(B) High-performance bicycles are likely to be improved more as a result of technological innovations developed in small workshops than as a result of technological innovations developed in major manufacturing concerns.
(C) Bicycle racers do not generate a strong demand for innovations that fall outside what is officially recognized as standard for purposes of competition.
(D) The technological conservatism of bicycle manufacturers results primarily from their desire to manufacture a product that can be sold without being altered to suit different national markets.
(E) The authorities who set standards for high-performance bicycle racing do not keep informed about innovative bicycle design.

作者: bananazoo    时间: 2011-12-2 00:28
答案:
逻辑链答案:DDCC(隐藏可见)
精练解析:
The argument concludes that a program instituted two years ago to increase
morale has ultimately caused the recent decrease in high school dropouts. You
must always recognize a causal conclusion when one is presented to you!
Whenever you encounter a causal conclusion, ask yourself if the relationship
must be as stated by the author or if another explanation can be found.
In simplified form, the conclusion appears as follows:
P = program to raise high school morale
RD = reduction in dropouts
CE
P -----> RD
Regardless of the question asked, this assessment is helpful. The question stem
asks you to weaken the argument, and according to the “How to Attack a
Causal Conclusion” section there are five main avenues of attack you should be
prepared to encounter. The correct answer, (A), falls into one of the most
frequently occurring of those categories—the alternate cause.
Answer choice (A): This is the correct answer. The answer attacks the
conclusion by introducing an alternate cause: it was not the morale program
that led to a decrease in high dropouts, but rather the fact that no jobs were
available for individuals contemplating dropping out of high school. The job
availability factor is important because the first sentence of the stimulus
indicates that high school students who drop out go to work. Thus, if a
recession led to a high level of unemployment, this could cause high school
students to rethink dropping out and stay in school.
Answer choice (B): At best, this answer confirms that some of the high school
students had a low morale, and in that sense, the answer strengthens the
argument. At worst, the answer choice is irrelevant.
Answer choice (C): The argument indicates that the dropout rate is lower
relative to the preceding year; there is no claim that the dropout rate ever
exceeded the retention rate. Thus, to suggest that more students stayed in school
than dropped out has no effect on the argument.
Answer choice (D): This is a Shell Game answer. The stimulus refers to high
school dropouts. This answer choice refers to high school graduates.
Answer choice (E): The argument uses information about the city’s overall
dropout rate. Therefore, the target high schools of the antidropout program are
irrelevant.
作者: fox0923    时间: 2011-12-2 00:31
我来啦~bana~~~
我来抢沙发~~~沙发成功!

精练-----------------25s---------------------weakenB: The high school dropout exists among high school students, and then they go to work.
P: However, the high school dropout has been decreased than the previous year.
C: The author concluded that the reason is because of the moral program taken a years ago has been effective in high school .
Prephrase: More students with higher moral minds entered high school.
                Other reasons can be brought in to weaken the conclusion, such as job market need, etc.
               
Analysis:
(A) There was a recession that caused a high level
of unemployment in the city.-----------------------------?YEAH!  If recession happened, then there wouldn't be a huge demand for the job market, then students would stay in high school.
(B) The morale of students who dropped out of
high school had been low even before they
reached high school.--------------------------------------We don't care if these students have low moral even before entered high school.
(C) As in the preceding year, more high school
students remained in school than dropped
out.-----------------------------------------------------------this basically restates the premise, so it doesn't make any sense.
(D) High schools in the city established placement
offices to assist their graduates in obtaining
employment.------------------------------------------------this is a sound approach, but this only brings alternative reason against the program stated in the argument. If the employment assistant is helpful, then there should be more dropout.
(E) The antidropout program was primarily aimed
at improving students’ morale in those high
school with the highest dropout rates.-----------------we don't care about the goal of this program.

作者: bananazoo    时间: 2011-12-2 00:32
搬个小板凳~排排坐~
作者: bananazoo    时间: 2011-12-2 01:41
background:High school students who feel that they are not succeeding in high school often drop out before graduating and go to work.
P:Last year, dropout rate was lower than the previous year’s rate.
C:a program instituted two years ago to improve the morale of high school students has begun to take effect to reduce dropouts.
预测:给出它因,别的原因导致了dropout rate 的下降,比如经济不景气什么的~
(A) There was a recession that caused a high level of unemployment in the city.
right~
(B) The morale of students who dropped out of high school had been low even before they reached high school.
这些人士气低不影响整体的提高,我们还是可以说政策有效~没有削弱
(C) As in the preceding year, more high school students remained in school than droppedout.
这,加强了,虽然没有说是这个政策的作用,但还是加强作用吧
(D) High schools in the city established placement offices to assist their graduates in obtaining employment.
其实也有给出它因的感觉,不过,这个原因的影响远没有A大,而且帮助毕业生找到工作,如果你不毕业也能找到,其实也不需要这些帮助,也还是会退学,没有削弱(bible说:这叫偷换概念~)
(E) The antidropout program was primarily aimed at improving students’ morale in those high school with the highest dropout rates.
这个,无关吧~只是说了这个政策的主要目的,没有削弱的意思~(bible:整体与部分概念不同,注意这些细节吧~慢慢体会)

看这里与开头的work有关~所以,我们要注意题中给出的细节~
作者: jssqzgr    时间: 2011-12-2 01:57
great!  got all correctly!
作者: fox0923    时间: 2011-12-2 05:51
bana好快~~~


作者: zz42050524    时间: 2011-12-2 10:18
精练: Weaken 36s
Students that are feel not succeed probably dropout before graduate. While last year the number of dropout students is lower than previous,
That’s the evidence that the morale encouragement of students is taking effect.
Pre: Some other reason cause students not dropout.
(A) There was a recession that caused a high level of unemployment in the city. 这个可以
(B) The morale of students who dropped out of high school had been low even before they reached high school. 这个说明不了学生dropout的原因
(C) As in the preceding year, more high school students remained in school than dropped out.
Irrelevant 留下的多少不是论述的内容
(D) High schools in the city established placement offices to assist their graduates in obtaining employment. 乍一看,有点像,仔细想下不是,自己都能找到工作的话,学校这个行为就没有吸引力。
(E) The antidropout program was primarily aimed at improving students’ morale in those high school with the highest dropout rates. Irrelevant

逻辑链:
2. weaken 30s
这个逻辑链里出过, 种植的农作物不同  D
3.  assumption 50s
P: Scientists enter their field and accept as their colleagues those with similar motivation.
C: Therefore, when scientists gain reputation as a expounder to general audiences, the scientists wouldn’t be their colleagues.
Pre: The colleagues must have similar motivation.
D  和推测有点出入,答案的重点放在了 research。
4. must be true
Two divisions of a company performed different in the last two years, A accounts for 20% sales and 40% profit, but B accounts for the balance.
A has a higher profit rates than B.
C
5. 这道题逻辑链也是出过  assumption  C
作者: xeyyxzty    时间: 2011-12-2 12:05
1.background information:.High school students who feel that they are not succeeding in high school often drop out before graduating and go to work. premise:the city’s high school dropout rate was significantly lower than the previous year’s rate.
conclusion:the program instituted two years ago to improve the morale of high school students has begun to take effect to reduce dropouts.
the program began two years ago, but it doesn't have effect until now~and is there any link between the morale and the dropouts?
(A) There was a recession that caused a high level of unemployment in the city.
--right
(B) The morale of students who dropped out of high school had been low even before they reached high school.

--irrelevant
(C) As in the preceding year, more high school students remained in school than dropped out.

--support
(D) High schools in the city established placement offices to assist their
graduates in obtaining employment.
--irrelevant
(E) The antidropout program was primarily aimed at improving students’ morale in those high school with the highest dropout rates.

--irrelevant
思路呀~
You must always recognize a causal conclusion when one is presented to you! Whenever you encounter a causal conclusion, ask yourself if the relationship must be as stated by the author or if another explanation can be found.
The correct answer, (A), falls into one of the most frequently occurring of those categories—the alternate cause.
2.background information: Neighboring Country Y has experienced the same climatic conditions, but while agricultural production has been falling in Country X, it has been rising in Country Y.
premise: its new government came to power.
conclusion:An overly centralized economy, not the changes in the climate, is responsible for the poor agricultural production in Country X
maybe crops are different~the style of an economy has nothing to do with the production of agriculture.
(A) Industrial production also is declining in Country X.
--irrelevant
(B) Whereas Country Y is landlocked, Country X has a major seaport.
--irrelevant
(C) Both Country X and Country Y have been experiencing drought conditions.
--irrelevant
(D) The crops that have always been grown in Country X are different from those that have always been grown in Country Y.
--right
(E) Country X's new government instituted a centralized economy with the intention of ensuring an equitable distribution of goods.
--irrelevant
3.background information:Generally scientists enter their field with the goal of doing important new research and accept as their colleagues those with similar motivation.  
premise: when any scientist wins renown as an expounder of science to general audiences
conclusion: most other scientists conclude that this popularizer should no longer be regarded as a true colleague.
the popularizers don't have the goal of doing importand new research
(A) serious scientific research is not a solitary activity, but relies on active cooperation among a group of colleagues
--irrelevant
(B) research scientists tend not to regard as colleagues those scientists whose renown they envy
--irrelevant???
(C) a scientist can become a famous popularizer without having completed any important research
--irrelevant
(D) research scientists believe that those who are well known as popularizers of science are not motivated to do important new research
--right
(E) no important new research can be accessible to or accurately assessed by those who are not themselves scientists
--irrelevant
4.premise:A company's two divisions performed with remarkable consistency over the past three years:  in each of those years, the pharmaceuticals division has accounted for roughly 20 percent of dollar sales and 40 percent of profits, and the chemicals division for the balance.conclusion:???
没有思路呢......
(A) Total dollar sales for each of the company's divisions have remained roughly constant.
--unknown
(B) The pharmaceuticals division has faced stiffer competition in its markets than has the chemicals division.
--unknown
(C) The chemicals division has realized lower profits per dollar of sales than has the pharmaceuticals division.
--right
(D) The product mix offered by each of the company's divisions has remained unchanged.
--unknown
(E) Highly profitable products accounted for a higher percentage of the chemicals division's sales than of those of the pharmaceuticals division.
--unknown
5.background information:The technological conservatism of bicycle manufacturers is a reflection of the kinds of demand they are trying to meet.  
premise:The only cyclists seriously interested in innovation and willing to pay for it are bicycle racers.  
conclusion: innovation in bicycle technology is limited by what authorities will accept as standard for purposes of competition in bicycle races
bicycle racers will only use bicycles in races or in training.
(A) The market for cheap, traditional bicycles cannot expand unless the market for high-performance competition bicycles expands.
--irrelevant
(B) High-performance bicycles are likely to be improved more as a result of technological innovations developed in small workshops than as a result of technological innovations developed in major manufacturing concerns.
--irrelevant
(C) Bicycle racers do not generate a strong demand for innovations that fall outside what is officially recognized as standard for purposes of competition.
--right
(D) The technological conservatism of bicycle manufacturers results primarily from their desire to manufacture a product that can be sold without being altered to suit different national markets.
--irrelevant
(E) The authorities who set standards for high-performance bicycle racing do not keep informed about innovative bicycle design.
--irrelevant
作者: 187167055    时间: 2011-12-2 12:14
新手上路~
作者: qiuhua01234567    时间: 2011-12-2 13:04
zhan
昨天和人干架去了,补上。
Weaken

BackgroundHigh school students who feel that they are not
succeeding in high school often drop out before
graduating and go to work

Premise: the program instituted
two years ago to improve the morale of high school
students has begun to take effect to reduce dropouts

Conclusion: the
city’s high school dropout rate was significantly
lower than the previous year’s rate.

(A)There was a recession that caused a high level
of unemployment in the city.

-------------------------------------------------------correct
(B) The morale of students who dropped out of
high school had been low even before they
reached high school.

----------------------------------------------------------irrelevent
(C) As in the preceding year, more high school
students remained in school than dropped
out.

-----------------------------------------------------------support
(D) High schools in the city established placement
offices to assist their graduates in obtaining
employment.

-----------------------------------------------------support
(E) The anti dropout program was primarily aimed
at improving students’ morale in those high
school with the highest dropout rates.

------------------------------------------------------support
2weaken

conclusion: An overly centralized economy, not the changes in the climate, is responsible for the poor agricultural production in Country X since its new government came to power.

Premise: Neighboring Country Y has experienced the same climatic conditions, but while agricultural production has been falling in Country X, it has been rising in Country Y.

(A)Industrial production also is declining in Country X.
(B) Whereas Country Y is landlocked, Country X has a major seaport.
(C) Both Country X and Country Y have been experiencing drought conditions.
(D) The crops that have always been grown in Country X are different from those that have always been grown in Country Y.

--------------------------------------------------------------------------------------------------correct
(E) Country X's new government instituted a centralized economy with the intention of ensuring an equitble distribution of goods.

Assumption

Background: Generally scientists enter their field with the goal of doing important new research and accept as their colleagues those with similar motivation

Premise: when any scientist wins renown as an expounder of science to general audiences,

Conclusion: most other scientists conclude that this popularizer should no longer be regarded as a true colleague.

(A)serious scientific research is not a solitary activity, but relies on active cooperation among a group of colleagues
(B) research scientists tend not to regard as colleagues those scientists whose renown they envy

--------------------------------------------------------------------------------------------------------correct
(C) a scientist can become a famous popularizer without having completed any important research
(D) research scientists believe that those who are well known as popularizers of science are not motivated to do important new research
(E) no important new research can be accessible to or accurately assessed by those who are not themselves scientists

A company's two divisions performed with remarkable consistency over the past three years:  in each of those years, the pharmaceuticals division has accounted for roughly 20 percent of dollar sales and 40 percent of profits, and the chemicals division for the balance

(A) serious scientific research is not a solitary activity, but relies on active cooperation among a group of colleagues
(B) research scientists tend not to regard as colleagues those scientists whose renown they envy
(C) a scientist can become a famous popularizer without having completed any important research
(D) research scientists believe that those who are well known as popularizers of science are not motivated to do important new research
(E) no important new research can be accessible to or accurately assessed by those who are not themselves scientists

Background: The technological conservatism of bicycle manufacturers is a reflection of the kinds of demand they are trying to meet.

Premise: The only cyclists seriously interested in innovation and willing to pay for it are bicycle racers.

Conclusion:innovation in bicycle technology is limited by what authorities will accept as standard for purposes of competition in bicycle races.

(A)The market for cheap, traditional bicycles cannot expand unless the market for high-performance competition bicycles expands.
(B) High-performance bicycles are likely to be improved more as a result of technological innovations developed in small workshops than as a result of technological innovations developed in major manufacturing concerns.
(C) Bicycle racers do not generate a strong demand for innovations that fall outside what is officially recognized as standard for purposes of competition.

---------------------------------------------------------------------------------------------------------correct
(D) The technological conservatism of bicycle manufacturers results primarily from their desire to manufacture a product that can be sold without being altered to suit different national markets.
(E) The authorities who set standards for high-performance bicycle racing do not keep informed about innovative bicycle design.



作者: winghyy    时间: 2011-12-2 13:26
精炼题
36s-weaken
This year, the dropout rate of last year is lower than that of the previous year.  This is an encouraging evidence that the program instituted 2 year ago take effect to reduce dropout.
Prephase: this year's rate is higher than the last year's.
选A

(A) There was a recession that caused a high level of unemployment in the city. ——correct [the alternate cause]
(B) The morale of students who dropped out of high school had been low even before they
reached high school. -strengthen
(C) As in the preceding year, more high school students remained in school than dropped out.
——irrelevant [students remained v.s. dropped out ]
(D) High schools in the city established placement offices to assist their graduates in obtaining
employment.  ——out of scope [graduates]
(E) The antidropout program was primarily aimed at improving students’ morale in those high
school with the highest dropout rates. ——out of scope [in those high school with the highest dropout rates]
作者: winghyy    时间: 2011-12-2 14:04
逻辑链
49. 36s-weaken
The agriculture production is lower in X than Y. While the climatic condition in X  and in Y is the same. Therefore, it is the centralized economy, not the climate condition, that cause the low agricultural production in X.
Prephase: Other causes——agricultral process.
选D

50. 50s-explain
Scientist enter their field with the goal of doing important new research and accept as their colleagues. Therefore, when a scientis become an expounder of science to general audience, most other scientists conclude that he is not a true colleague.
Prephase: the popularizers don't do any important new research.
选D

51. 35s-infer
The P division accounts for 20% of sales and 40% of profits. The D divison for the balance.
选C

52. 50s-assumption
The innovation of bicycle techonolgy is limited by authorities. The only cyclists seriously interested in innovation and willing to pay for it are bicycle racers.
Prephase: authorities' requirements affect bicycle racer's buying decision.
选C
作者: ugly5552000    时间: 2011-12-2 23:35
1、Background: those high school students drop out before graduation because they feel they are not successful in high school. Last year, the city’s high school dropout rate was impressively lower than the previous year.
Premise: it is the program instituted two years ago to improve the morale of high school students has began to take effect to reduce dropouts.
Prephrase: if the rural area’s high school students are counted in, the general dropout ratio does not decline. Many companies has risen their criteria of recruitment that the applicants must have graduated from high school.

A
A: correct answer, the recession will reduce the working opportunities, the high school students must contemplate the outcome of dropout.
B: irrelevant
C: irrelevant
D: if the assistance is useful, it will cause more dropouts
E: irrelevant
2、Background: the agriculture production in Country X has been falling while that in Country Y has been rising.
Premise: because Country X and Y is under the same climatic condition, so the agriculture production fall in Country X is probably caused by the overly centralized economy.
Prephrase: many peasants leave their land and get into cities to work as immigrant workers.

D
A: irrelevant
B: what the territory of X and Y is like does not make any sense.
C: then it is hard to explain why agriculture production still rise under drought condition.
D: the different crops of X and Y make that even under same climatic condition, the agriculture productions might be different.
E: irrelevant
3、Background: generally speaking, only scientists with the goal of important new research could be accepted as colleagues by other scientists in the same field.
Premise: when any scientist wins popularity by explaining science to general audience, he is no longer be regarded as a true colleague.
Prephrase: when a scientist wins popularity, his concern of work will no longer be the import research.
D
A: nothing about cooperation
B: envy makes no sense
C: the point is not how to be a popularizer, but what will happen after being a popularize
D: when the important new research cannot be completed, a scientist loses one of the conditions to be accepted as colleague.
E: irrelevant
4、Background: a company has two divisions, the pharmaceuticals division and the chemicals division. In the last three years, the pharmaceuticals division accounted for roughly 20% of sales and 40% of profits, and the chemicals division takes roughly 80% of sales and 60% of profits.
Prephrase: the pharmaceuticals division is more profitable than the chemical division.
C
A: irrelevant
B: the one which faces stiffer competition should be chemical division
C: the choice is the answer
D: irrelevant
E: cannot be concluded
5、Background: the technological innovation of bicycle is closely connected with the demands of bicycle. Only the bicycle racers are willing to pay for the innovation in bicycle.
Premise: so the innovation in bicycle technology is limited by the authorities of bicycle races.
C
A: irrelevant
B: irrelevant
C: without the acknowledgement of the racing authorities, the racer will not demand innovations
D: irrelevant
E: irrelevant
作者: balapupu    时间: 2011-12-3 15:36
1.[23s]
P: Students who feel that they can not success in the high school will drop out and work?Last year, the school drop out rate decrease?C: this suggest that the measurement to improve the students moral standard has effect.
Weaken: because the bad economic, students drop out school can not find job so they stay at school.
A:
(A) There was a recession that caused a high level
of unemployment in the city.-->R
(B) The morale of students who dropped out of
high school had been low even before they
reached high school.-->support
(C) As in the preceding year, more high school
students remained in school than dropped
out.-->irrelevant comparison.
(D) High schools in the city established placement
offices to assist their graduates in obtaining
employment.-->support
(E) The anti dropout program was primarily aimed
at improving students’ morale in those high
school with the highest dropout rates.-->support
作者: balapupu    时间: 2011-12-3 16:24
逻辑链:
1.[18s]
P: the neighbor country Y has the same climate with the X, however the agriculture in Y increase while X decrease.
C: Over centralized economy lead to the poor agriculture of X.
Weaken: the soil of the agriculture in two different is different..
A:
(A)    Industrial production also is declining in Country X.-->irrelevant with other industry
(B) Whereas Country Y is landlocked, Country X has a major seaport.-->irrelevant with the kinds of the city
(C) Both Country X and Country Y have been experiencing drought conditions.-->support
(D) The crops that have always been grown in Country X are different from those that have always been grown in Country Y.-->R
(E) Country X's new government instituted a centralized economy with the intention of ensuring an equitable distribution of goods.-->irrelevant.
2.[17s]
P: Scientists enter the field with goal of doing important new research and accept the college with the same motivation?if some scientists win the renown?it is thought that they are not true college.
Assumption: true college did not work for win the renown.
A:
(A)    serious scientific research is not a solitary activity, but relies on active cooperation among a group of colleagues?irrelevant wit the serious
(B) research scientists tend not to regard as colleagues those scientists whose renown they envy?no mention about the envy.
(C) a scientist can become a famous popularizer without having completed any important research-?not mention.
(D) research scientists believe that those who are well known as popularizers of science are not motivated to do important new research?R
(E) no important new research can be accessible to or accurately assessed by those who are not themselves scientists ?irrelevant with the people not scientists.

3.数字比例关系,做过了。
4.【15s】
P:the technology of the bicycle reflect the consumer needs?the only consumer interested in the innovation of the bicycle is the racers?so the innovation technology fill into the standard of competition purpose.
Assumption: racers have no purpose without the competition.
A:
(A) The market for cheap, traditional bicycles cannot expand unless the market for high-performance competition bicycles expands.-->irrelevant with the expands
(B) High-performance bicycles are likely to be improved more as a result of technological innovations developed in small workshops than as a result of technological innovations developed in major manufacturing concerns.-->irrelevant with the factory size.
(C) Bicycle racers do not generate a strong demand for innovations that fall outside what is officially recognized as standard for purposes of competition.-->R
(D) The technological conservatism of bicycle manufacturers results primarily from their desire to manufacture a product that can be sold without being altered to suit different national markets.-->not mention in the passage.
(E) The authorities who set standards for high-performance bicycle racing do not keep informed about innovative bicycle design?irrelevant with the information.
作者: monica3m    时间: 2011-12-3 19:08
II, background: students drop out rate is high
 remise: program that is mentioned to improve the morale the high school students.
 Conclusion: the program is effective to reduce the students' drop out rate.

49, background: climate changes
  Premise: Country X and Y suffered from the same climatic conditions.
  Conclusion: agricultural production has been falling in country X, but it has been rising in country Y.
D
50, Background: scientists' goal is doing important new research and is accepted by their colleagues.
  Premise: some of scientists are known as expounders of science to general audiences.
  Conclusion: popularizer should no longer be regarded as a true colleague.
D
51,  Background: department P and C performed with remarkable consistency over the past three years.
  Premise: Department P = 20% sales/ 40% profits.
         Department C = 80% sales/ 60% profits
  Conclusion:
C
新手上路~
作者: 风无衣    时间: 2011-12-3 23:38
【精炼1-13】
weaken_37s
background information:high school students who think they are not successful often dropout from school before graduate and go to work.
premise:last year, the dropout rate was lower than previous year's rate
conclusion:it is obvious the program which instituted to improve morale of high school that took effect to reduce the dropout rate.
(A) There was a recession that caused a high level
of unemployment in the city.——irrelevant
(B) The morale of students who dropped out of
high school had been low even before they
reached high school.——have no effect on this issue
(C) As in the preceding year, more high school
students remained in school than dropped
out.——has no effect on this issue
(D) High schools in the city established placement
offices to assist their graduates in obtaining
employment.——correct.他因导致。
(E) The antidropout program was primarily aimed
at improving students’ morale in those high
school with the highest dropout rates.——irrelevant about the purpose of the program

哟西~做错了~果然几天没看CR报应啊~呜呜~

The argument concludes that a program instituted two years ago to increase
morale has ultimately caused the recent decrease in high school dropouts. You
must always recognize a causal conclusion when one is presented to you!
Whenever you encounter a causal conclusion, ask yourself if the relationship
must be as stated by the author or if another explanation can be found.
In simplified form, the conclusion appears as follows:
P = program to raise high school morale
RD = reduction in dropouts
CE
P -----> RD

Answer choice (A): This is the correct answer. The answer attacks the
conclusion by introducing an alternate cause: it was not the morale program
that led to a decrease in high dropouts, but rather the fact that no jobs were
available for individuals contemplating dropping out of high school.
The job
availability factor is important because the first sentence of the stimulus
indicates that high school students who drop out go to work.
Thus,if a
recession led to a high level of unemployment, this could cause high school
students to rethink dropping out and stay in school.

Answer choice (D): This is a Shell Game answer. The stimulus refers to high
school dropouts.
This answer choice refers to high school graduates.
当时做题其实没有觉得D是偷换概念来着~
我觉得高校帮助学生就业,就使得打算早辍学的同学发现留在学校的好处了噻~所以他们就留在学校的多了~现在看来应该是我YY得过于深入了~哎。。。CR我该拿你怎么办。。。

作者: 珍妮弗    时间: 2012-3-8 10:06
xie
作者: Threesu    时间: 2012-4-1 19:09
T:19
P:Students without certain morale will try to drop out and go
to work. The rate of the students drop out in high school is
decreasing.
C:Something was done is helpful in reducing such things.
A
T:32
P:Suffering the same climitic condition, X experienced agriculture
reduce and Y did not.
C:The economy is responsible for this phenomethon.
D
T:34
P:The colleagues of scientists are the ones that enjoy the same
objection of doing researches.
C:The popularizer who obtain the renown of introducing science
to general audience are no longer regarded as colleagues of them.
D
T:23
P:Two divisions performed consistency in the past three years,
P did 20 percent of sales and 40 percent profit,
C

T:25
P:The people who are in favor of bicycles are racers.
C:The innovation will be limited.
作者: leewonting    时间: 2012-4-11 11:00
69
High school students who feel that they are not succeeding in high school often drop out before graduating and go to work. Last year, however, the city’s high school dropout rate was significantly lower than the previous year’s rate. This is encouraging evidence that the program instituted two years ago to improve the morale of high school students has begun to take effect to reduce dropouts.
Which one of the following, if true about the last year, most seriously weakens the argument?

(A) There was a recession that caused a high level of unemployment in the city.
(B) The morale of students who dropped out of high school had been low even before they reached high school.(在进入高中之前,学生的士气就已经很低了,那么辍学率的减低,有可能就是项目领导学生士气提高了,support)
(C) As in the preceding year, more high school students remained in school than dropped out.
(D) High schools in the city established placement offices to assist their graduates in obtaining
employment.
(E) The antidropout program was primarily aimed at improving students’ morale in those high school with the highest dropout rates.
背景:High school students who feel that they are not succeeding in high school often drop out before graduating and go to work.
条件:Last year, the high school dropout rate was significantly lower than the previous year’s rate.
结论:The program instituted two years ago to improve the morale of high school students has begun to take effect to reduce dropouts.
推测:how about the students who are likely to drop out did not enter the high school? Or how about other reasons such as the need of market?
选B,(A is confusing, whether the unemployment in the city will encourage the students to finish their schooling?)
应该选A!!!!!!!!!!!!!!!!!!!!!!!!!!!!!!!!!!!!!!!!!
作者: leewonting    时间: 2012-4-11 11:00
背景:The climate in X is the same as that of Y.
条件:The agriculture falls in X while rising in Y
结论:An overly centralized economy, not the changes in climates, is responsible for the poor agriculture production in Country X.
推测:is there any differences in the soils, water? Or the crops?
选D (C is confusing, I had chosen this one, though the geography roles are different, it cannot deny if the economy takes effect)


70
背景:Generally scientists enter their field with the goal of doing important new research and accept as their colleagues those with similar motivation.
条件:When any scientist wins renown as an expounder of science to general audiences, most other scientists conclude that this popularize should no longer be regarded as a true colleague
结论:explain this situation
推测:一旦是对普罗大众做科学简介的,一律认为不是真正的科学同事
选D(之前选错过)


71
背景:A company’s two divisions performed with remarkable consistency over the past 3 years.
条件:The pharmaceuticals division has accounted for roughly 20% of dollar sales and 40% of profits while the chemicals division has accounted for 80% of dollar sales and 60% of profits
结论:Pharmaceuticals is better than chemicals
推测:like above.
选C


72
背景:The technological conservatism of bicycle manufacturers is a reflection of the kinds of demand they are trying to meet.
条件:Only cyclists seriously interested in innovation and willing to pay for it are bicycle racers.
结论:Innovation in bicycle technology is limited by what authorities will accept as standard for purposes of competition in bicycle races
推测:C
作者: Roxas605    时间: 2012-4-11 11:29
这些都做过。   = = 期待新题~
作者: Rena张    时间: 2012-4-18 21:53
1.
Time: 1'03''
Premise: high school students dropout rate was high. They go to work. However, last year the rate lowered significantly.
Conclusion: the program instituted 2years ago has begun to take effect to reduce dropouts.
Type: weaken
Prephrase: high unemployment  
Choice: A
2. 41''   D
3. 1'25''  D
4. 30''  C
5. 42''  C
--------------
怎么做的这么慢呢~
作者: FB小贝    时间: 2012-5-9 15:31
2. High school students who feel that they are not
succeeding in high school often drop out before
graduating and go to work. Last year, however, the
city’s high school dropout rate was significantly
lower than the previous year’s rate. This is
encouraging evidence that the program instituted
two years ago to improve the morale of high school
students has begun to take effect to reduce dropouts.
Which one of the following, if true about the last
year, most seriously weakens the argument?
----------Cause and Effect Problems NO.2
Premise: not succeed – drop out
Dropout rate was lower than the previous year’s rate.
Conclusion: the program instituted two years ago to improve the morale of high school students begun to take effect
Weaken: another reason/

(A) There was a recession that caused a high level
of unemployment in the city.
correct
(B) The morale of students who dropped out of
high school had been low even before they
reached high school.
Strengthen
(C) As in the preceding year, more high school
students remained in school than dropped
out.
Rate and number.  incorrect
(D) High schools in the city established placement
offices to assist their graduates in obtaining
employment.
irrelevant
(E) The antidropout program was primarily aimed
at improving students’ morale in those high
school with the highest dropout rates.
irrelevant


最近几天精练题做得很好~
作者: dwindwin1106    时间: 2012-5-13 06:06
(1)P:HS students who feel that they are not succeeding in HG often drop out and go to work.
Past year, the HG dropout rate was significantly lower.
C:the program instituted 2 years ago has begun to take effect to reduce dropouts
weaken: other reasons account for the decreasing dropouts
Answer: A
A.Correct ------ 由于unemploymentdropouts难以找到工作,只能继续学习,从而降低了dropout rate
B.Irrelevant ------ 之前的moral如何与现在无关
C.Irrelevant ------ 过去的dropout情况与现在无关
D.Irrelevant ------ graduates与文章讨论对象不符
E.Strengthen ------ 既然program针对dropout rates最高的HG,那现在dropout rate降低了,说明program是有一定作用的


(2)P:the same climatic conditions, agricultural production in X falls, but agricultural production in Y rises
C:an overly centralized economy, not the changes in the climate, is responsible for the poor agricultural production in X since its new government came to power.
Weaken:种植的作物不同?
Answer: D
A.Irrelevant
B.irrelevant
C.Irrelevant
D.Correct
E.Irrelevant


(3)p:generally sci enter their fields with the goal of doing important new research & accept those with similar motivation as their colleagues
C:when sci is an expounder of science to general audience, most other sci conclude that this popularizer should no longer be regarded as a true colleague.
Assumption: popularizer does not have the goal of doing important new research
Answer: D
A.irrelevant
B.irrelevant
C.irrelevant
D.correct
E.irrelevant


(4)F division account for 20% sales&40% profits
F:C division account for 80% sales&60% profits
must be true
Answer: C
A.Irrelevant
B.irrelevant
C.Correct
D.Irrelevant
E.Irrelevant


(5)P:the technological conservatism of bicycle manu. Reflects the demand they need to meet.
P:the only cyclists seriously interested in innovation and willing to pay for it are bicycle racers.
C:innovation in bicycle tech is limited by what authorities will accept as standard for competition in bicycle races.
Assumption: racers need to buy bicycles that the authorities accept as standard?
Answer: C
A.Irrelevant
B.Irrelevant
C.Correct ------ racers除了官方认定的innovations之外,不会过多的产生其他要求,这样官方就起了主要作用,使得结论成立
D.Irrelevant
E.irrelevant
作者: spencerX    时间: 2012-5-21 07:13
【精练】
background: 学生如果觉得自己在学校不成功,就会辍学并去工作。
premise:辍学的学生人数减少;实行了一个提升学生morale的program
conclusion:是因为program使得辍学的人数减少
pre:找不到工作,只能在学校读书(他因)
Analysis:
A.    Correct。因为辍学→工作,如果(非工作),那么(非辍学),则辍学率自然就下降了。
B.    Irrelevant
C.    Out of scope
D.    Irrelevant
E.    Irrelevant

【逻辑链】
1.    remise:X和Y有相同的天气,但是Y的收成比X好
Conclusion:造成X收成差的原因的过度集中的经济(政府)
Pre:X和Y不能进行类比(有本质的不同)
My answer:D
2.    remise:Generally scientists enter their field with the goal of doing important new research and accept as their colleagues those with similar motivation.
Conclusion:when any scientist wins renown as an expounder of science to general audiences, most other scientists conclude that this popularizer should no longer be regarded as a true colleague
Pre:一个scientist如果popularize了,那么就不是with the goal of doing important new research (fill the gap)
My answer: D
3.    Fact: P-20%sale;40%profit
C-80%sale;60%profit
Pre: 40%/20%=2
   60%/80%=3/4
My answer: C
4.    Background: The technological conservatism of bicycle manufacturers is a reflection of the kinds of demand they are trying to meet
Premise: The only cyclists seriously interested in innovation and willing to pay for it are bicycle racers.
conclusion:Innovation in bicycle technology is limited by what authorities will accept as standard for purposes of competition in bicycle races.
Pre:cyclists will only interested in innovations that are permitted by authorities
My answer: C
作者: baseboss    时间: 2012-5-26 21:42
1.Students who can not study well will dropout and go to work.
presume:
Artribute the dropout rate to project that improve  morale of the students is not sufficient.
Dropout rate low maybe due to students can not find the job.

(A) There was a recession that caused a high level
of unemployment in the city.
-->Right
(B) The morale of students who dropped out of
high school had been low even before they
reached high school.
-->irrelevant
(C) As in the preceding year, more high school
students remained in school than dropped
out.
-->support
(D) High schools in the city established placement
offices to assist their graduates in obtaining
employment.
-->irrelevant
(E) The antidropout program was primarily aimed
at improving students’ morale in those high
school with the highest dropout rates.
-->irrelevant.


2 CompareX and Y,same climic condition,but Y than X,so a overly centralized economic is the responsible factor.
presume:
Not efficient.
Y also a overly centralized economic
D

3 Only the scientist do the new research can be respect as scientist.in contract the popularizers scientist is not.
presume:
popularizers scientist do not do the important new research.

D

4 p 20% sales + 40 % profit  vs  c  balance
==> P + C = consistency  o or 5
p has more profit than C
C

5 bike manufacture innovation is depend on the buyer's demand.
Guy who are care about the innovation of bike is bike racer.
-->bike racer's demand on the bike innovation is limitted by  authorities' standard.
C
作者: Threesu    时间: 2012-6-7 00:31
B: if students feel frustrated, they would drop out school
P: the dropping out rate is significantly decreased
C: the moral education is working

(A) weaken
(B) support
(C) As in the preceding year, more high school
students remained in school than dropped
out.
(D) weaken
(E) support
Shell game 题目中是high school dropouts,这里是high school graduates

B: In the same climate situation, production is raising in Y but decreasing in X
P: the are in the same climate
C: the factor is overly centralized agriculture

(A) Industrial production also is declining in Country X.
(B) Whereas Country Y is landlocked, Country X has a major seaport.
(C) Both Country X and Country Y have been experiencing drought conditions.
(D) weaken
(E) irrelevant

B: scientists enter the field to do the research and accepted as college with the people who have the same motivation
P: if one is well known as an expander he will no longer be accepted as college

(A) serious scientific research is not a solitary activity, but relies on active cooperation among a group of colleagues
(B) research scientists tend not to regard as colleagues those scientists whose renown they envy
(C) a scientist can become a famous popularizer without having completed any important research
(D) research scientists believe that those who are well known as popularizers of science are not motivated to do important new research
(E) no important new research can be accessible to or accurately assessed by those who are not themselves scientists
其他都不相关

B: the two divisions work consistently in the past three years
P: the P has accounted for 20 percent of sales and 40 percent of profit
C: the chemicals for the balance

(A) Total dollar sales for each of the company's divisions have remained roughly constant.
(B) The pharmaceuticals division has faced stiffer competition in its markets than has the chemicals division.
(C) The chemicals division has realized lower profits per dollar of sales than has the pharmaceuticals division.
(D) The product mix offered by each of the company's divisions has remained unchanged.
(E) Highly profitable products accounted for a higher percentage of the chemicals division's sales than of those of the pharmaceuticals division.

B: the only ones who demand the innovation of bicycle is bicycle racers.
C: the innovation in bicycle technology is limited by the accepting standard in the competition
(A)IRRELEVANT
(B) High-performance bicycles are likely to be improved more as a result of technological innovations developed in small workshops than as a result of technological innovations developed in major manufacturing concerns.
(C) Bicycle racers do not generate a strong demand for innovations that fall outside what is officially recognized as standard for purposes of competition.
(D) The technological conservatism of bicycle manufacturers results primarily from their desire to manufacture a product that can be sold without being altered to suit different national markets.
(E) The authorities who set standards for high-performance bicycle racing do not keep informed about innovative bicycle design.
作者: 二楼往下掉    时间: 2012-6-8 23:39
1.
(1)27s weaken
(2)逻辑链
Background:
High school students who feel that they are not succeeding in high school often drop out before graduating and go to work.
Primes:
Last year, however, the city’s high school dropout rate was significantly lower than the previous year’s rate.
Conclusion:
The program instituted two years ago to improve the morale of high school students has begun to take effect to reduce dropouts.
(3)先后发生,不为因果
(4)A:提出了新的因,即失业率高使得退学率降低,contender
B:没有提出新的解释,loser
C:没有对退学率降低做出新的解释,loser
D:学校设立了帮助学生就业的办公室,帮助各种学生找工作,对于在学校中不成功的学生没有特殊的帮助,应该不会对学生是否退学有影响,loser——这里很犹豫啊,当时!!!!
E:说明项目很有针对性,support,loser
所以,选A

2.
(1)24s weaken
(2)逻辑链
Primes:
Neighboring Country Y has experienced the same climatic conditions, but while agricultural production has been falling in Country X, it has been rising in Country Y.
conclusion:
An overly centralized economy is responsible for Country X’s poor agricultural production.
(3)错误类比,XY有很多不同的地方啊,没的可比
(4)A:证明经济政策不好啊,support,loser
B:一个内陆,一个沿海,没有可比性,contender
C:都经历了干旱天气,说明就是X的经济政策不好,support,loser
D:东西不一样,没有可比性,contender
E:不管政府的经济政策出于什么目的,都是产量下降了,无关选项,loser
B只是说道两个地方的地理位置不同,太宽泛了,而D具体说明了X与Y种的玉米品种不一样,所以受天气的影响也不一样,所以不一定是经济政策使得X的产量下降,所以选D

3.
(1)57s assumes
(2)逻辑链
Background:
Generally scientists enter their field with the goal of doing important new research and accept as their colleagues those with similar motivation.
(4)科学家将:向大众讲授=非同僚——>不讲授=同僚
A:科学不应该是孤独的活动,应该是一部分人的活动,loser
B:重点不在是否renown,loser
C:无关选项,popularizer 跟research的水平没关系,loser
D:科学家认为讲授的人的动机不是为了科学,而是为了出名,contender
E:无关选项,loser
所以,选D

4.
(1)25s must be true
(2)逻辑链
                                  Sales      rofits
Pharmaceuticals         20%      40%
chemicals                    80%       60%
(4)A:每个部门的销售量占重量是基本不变的,但是不能说明销售量就没变,百分数不等于总量,loser
B:argument没提到竞争对手的问题,该选项意图通过说竞争对手来解释P部门的销售量占的少的问题,但竞争对手只是一个原因,loser
C:C部门虽然销售量很大,但是利润的占比却没有销售量这么大,也就是说C部门买的东西没有P部门的之前,contender
D:原文没提到,loser
E:没有区分高低利润的产品,loser
所以,选C

5.
(1)20s assumption
(2)逻辑链
Primes:
The only cyclists seriously interested in innovation and willing to pay for it are bicycle racers.
Innovation in bicycle technology is limited by what authorities will accept as standard for purposes of competition in bicycle races.
Conclusion:
The technological conservatism of bicycle manufacturers is a reflection of the kinds of demand they are trying to meet.
(4)A:跟cheap,traditional的车没关系,loser
B:文中没有出现该选项中提到的比较,loser
C:运动员对于非官方需要的科技改革是没有强烈需求的,说明了自行车科技保守一种对运动员需求的满足,contender
D:没有提到关于市场适应的问题,loser
E:无关选项,文中说科技革新以需求为导向,E中没有提到相关信息,loser
所以,选C
作者: jetyxo    时间: 2012-6-16 20:13
1  25"  background:high school students who fell unsuccessful always drop out and go to work
          premise: last year the rate of students drop out declined
          conclusion: the methods used to improve morale of students have take effect
         weaken: there were other reason stop students to drop out--->A recession reduce the rate of drop out, because students who drop out school can not get a job easily
------------------------------------------------------
2  22''  conclusion:iin X, overly centralized economic leads to poor agricultural production
           reason: the climat is the same in X as in Y,but  agricultural production in X decrease whereas increase in Y
          weaken: may be there was other difference  leads to these condition
        A  contrary to  premise; B --->may be weaken; C --suppot; E irrelevant    
        answer: D
-----------------------------------------------------------
3 27"  background: scientist enter a science fiel with goal of doing important new research  and accept people with the same goal
         conclusion: when a scientist become popular , other scientist will not treat him as a colleague
         assumption: those scientist who become popular will lose  their goal of doing important research---->D
--------------------------------------------------------------
4 17" background: two department:pharmaceuticals division ---20% sales   40% profits
                                                       chemical  division-----80% sales  60% profits
        what we can deduce from these data?------> pharmaceutical division is more profitable than chemical division--->C
---------------------------------------------------------------
5  30"  background: the bicycle innovation conservatism --->market demand
          premise: the only demand in technological innovation is from the bycicle racers
          conclution: bicycle innovation is limited by the bycicle racers' demand
           assumption: the racers' demand on innovation are conservatism--->C
作者: jiajiajudy    时间: 2012-6-29 04:24
1. BG: HS feel not succeeding often drop out before graduating and go to work
Improve that morale begun to effect ?dropout rate lower than previous year’s rate
(A) There was a recession that caused a high level of unemployment in the city.
(B) Paradox
(C) paradox
(D) irrelevant
(E) irrelevant
2. BG: CE( not changes in the climate) responsible for the poor agriculture in X
Premise: Y has experienced the same climate, but production rising while X diminishing
(A) irrelevant
(B) irrelevant
(C) irrelevant
(D) The crops that have always been grown in Country X are different from those that have always been grown in Country Y.
(E) irrelevant
3. BG: scientist enter field with goal of doing important new research & accept as their colleagues those with similar motivation
Conclusion: when renown? no longer as a true colleague
A) serious scientific research is not a solitary activity, but relies on active cooperation among a group of colleagues
(B) irrelevant
(C) irrelevant
(D) irrelevant
(E) irrelevant
4. BG: two divisions performed with remarkable consistency
Premise: pharmaceutical 20% sales, 40% profits
            Chemical balance
(A) Total dollar sales for each of the company's divisions have remained roughly constant.
(B) irrelevant
(C) irrelevant
(D) irrelevant
(E) irrelevant
5. BG: tech conservatism of bicycle manufacturers is a reflection of demand they meet
Premise: innovation & willing to pay are bicycle racers
Conclusion: tech is limited by the accept of authorities for purposes of competition
(A) irrelevant
(B) irrelevant
(C) Bicycle racers do not generate a strong demand for innovations that fall outside what is officially recognized as standard for purposes of competition.
(D) irrelevant
(E) irrelevant
作者: angelafeng    时间: 2012-7-3 15:53
罪过啊,好几天没做逻辑了。。。
【精练】
2. 30s

Background informationHigh school students who feel that they are not succeeding in high school often drop out before graduating and go to work.

PremiseLast year, however, the city’s high school dropout rate was significantly lower than the previous year’s rate.

ConclusionThis is encouraging evidence that the program instituted two years ago to improve the morale of high school students has begun to take effect to reduce dropouts.
prephrase
:(weaken);last year, the city’s joblessness leaded to high students prefer to stay in school rather than find a job.

AnswerA


(A) There was a recession that caused a high level
of unemployment in the city.------------correct
(B) The morale of students who dropped out of
high school had been low even before they
reached high school.-------------this cannot weaken the argument
(C) As in the preceding year, more high school
students remained in school than dropped
out.------------------------this has no relation with the dropout rate.
(D) High schools in the city established placement
offices to assist their graduates in obtaining
employment.---------------irrelevant
(E) The antidropout program was primarily aimed
at improving students’ morale in those high
school with the highest dropout rates.------------------support

【逻辑链】
49.41s
premise :  Neighboring Country Y has experienced the same climatic conditions, agricultural production has been falling in Country X, it has been rising in Country Y.

Conclusion: An overly centralized economy, not the changes in the climate, is responsible for the poor agricultural production in Country X since its new government came to power.
prephrase: (weaken) country x plant different crops from country Y’s.
answer



50.30S
premise :Generally scientists enter their field with the goal of doing important new research and accept as their colleagues those with similar motivation.  

Conclusion: when any scientist wins renown as an expounder of science to general audiences, most other scientists conclude that this popularizer should no longer be regarded as a true colleague.
prephrase: (assume)

Answer

51. 25S
premise: A company's two divisions performed with remarkable consistency over the past three years:  in each of those years, the pharmaceuticals division has accounted for roughly 20 percent of dollar sales and 40 percent of profits, and the chemicals division for the balance.
answer:C



52. 26S
premise: The technological conservatism of bicycle manufacturers is a reflection of the kinds of demand they are trying to meet.  The only cyclists seriously interested in innovation and willing to pay for it are bicycle racers.

Conclusion: innovation in bicycle technology is limited by what authorities will accept as standard for purposes of competition in bicycle races.
prephrase: (assumption)

Answer:C

作者: LuckyYolandaLi    时间: 2012-7-5 21:16
1.
1)计时:38s
2)逻辑链:
Background information: High school students who feel that they are not succeeding in high school often drop out before graduating and go to work.
Premise: Last year, the city’s high school dropout rate was significantly lower than the previous year’s rate.
Conclusion: the program instituted two years ago to improve the morale of high school students has begun to take effect to reduce dropouts.
3)推测:If the program would take effect, it would do as soon as it was instituted
4)选项分析:选A
A: Correct.不太确定,the students who want to drop out of high school cannot find a job, so they remain in school.
B: Irrelevant.
C: Irrelevant.
D: Irrelevant.
E: Irrelevant.

2.
1)计时:
2)逻辑链:
Premise:  Neighboring Country Y has experienced the same climatic conditions, but while agricultural production has been falling in Country X, it has been rising in Country Y.
Conclusion: An overly centralized economy, not the changes in the climate, is responsible for the poor agricultural production in Country X since its new government came to power.
3)推测:The same climatic conditions can take different effects in different countries.
4)选项分析:选D
A: Irrelevant或strengthen.
B: Irrelevant。。。有点犹豫
C: Strengthen.
D: Correct.
E: Irrelevant.

3.
1)计时:45s
2)逻辑链:
Premise: Generally scientists enter their field with the goal of doing important new research and accept as their colleagues those with similar motivation.
Conclusion: when any scientist wins renown as an expounder of science to general audiences, most other scientists conclude that this popularizer should no longer be regarded as a true colleague.
3)推测:Scientists who have won renown will no longer have a goal of doing important new research.
4)选项分析:选D

4.
1)计时:25s
2)逻辑链:
Premise: A company's two divisions performed with remarkable consistency over the past three years:  in each of those years, the pharmaceuticals division has accounted for roughly 20 percent of dollar sales and 40 percent of profits, and the chemicals division for the balance.
3)推测:The chemicals division has accounted for roughly 80 percent of dollar sales and 60 percent of profits in each of those three years.
4)选项分析:选C
A: Only two divisions have been mentioned in the passage. Maybe the company has other divisions.
C: Correct.

5.
1)计时:45s
2)逻辑链:
Background information: The technological conservatism of bicycle manufacturers is a reflection of the kinds of demand they are trying to meet.
Premise: The only cyclists seriously interested in innovation and willing to pay for it are bicycle racers.
Conclusion: Innovation in bicycle technology is limited by what authorities will accept as standard for purposes of competition in bicycle races.
3)推测:The cyclists are willing to pay for innovation which can make their bicycle fit the competition more.
4)选项分析:选C
作者: cleotina    时间: 2012-7-6 00:26
45''
weaken
b: 1,if students feel they will not success in high school often drop out
  2, last year the drop out rate decrease a lot compare with before
P: it is because the peogram to inprove the morale of hith school studem.
guessther factors lead to this result for example: the examtion is easier than before
OA : A
BCD: I
E:strong

35''
weaken
B: country X and country y with the same climate condition. the agriculture production in X decline since the new government came to power while the production in Y are rising.
C: it because the overlu centralized economy since the new governor
guess: x cultivate more kind of agriculture and Y only focus on one.
OA: D
ABC: Irrelevant
E: strong

40''
assume
B: 1.scients with the aim to doing important new research accept their colleague with the similar motivation.
  2 if a scient win the reputation from repeat others theory and show in the public
C: for this person they will no longer concern him a true colleague
Guess: they have no energy to focus on the new study
      they tend to fulfill with the superficial seccess
OA
a: weaken, but not concern with the condition 1
others: Irrelevant


30''
infer
1: two departments in one company
2: P account for 20 % sales and 40% perfits. the other  for the balance
Guess: the per sales has a higher profits in department P
OA: C
B: infer too much need the imformation out of the scope
others: wrong

38''
1: innovation in order to meet the consumer's need
2: the only group the innovation is the racer
C: innovation should focus on the purpose of compitation  
guess: assume that the racer's all need is compitation, mayby they also like bycicly in the daily life
OA: c
others: irrelevant
作者: Donts    时间: 2012-7-16 10:00
1.计时34”
premise:students who think themselves as failed in high school will drop out and go to work. two years ago, a program was insitituted to help solve this problem. the rate of dropout decrease.conclusion:the program begins to take effect.
推测:别的原因造成rate decrease/rate decrease和program之间没有联系
(A) There was a recession that caused a high level
of unemployment in the city.无法找到工作,所以回来读书,别的原因导致。正确。
(B) The morale of students who dropped out of
high school had been low even before they
reached high school.与现象舒缓没有关系。
(C) As in the preceding year, more high school
students remained in school than dropped
out.只是paraphrase了其中一个premise
(D) High schools in the city established placement
offices to assist their graduates in obtaining
employment.虽然有点像A选项,但是它的前提是,工作是这些学生辍学的理由,所以一旦工作有保障,学生就愿意读书,其实不然。而A的合理在于它没有说就业是学生辍学的理由,反之只是说学生找不到工作,只好回到学校。
(E) The antidropout program was primarily aimed
at improving students’ morale in those high
school with the highest dropout rates.与program实施的学校范围无关。
选择A
总结:觉得自己对D的排除方法比bible的要好。
作者: TICKCOCK    时间: 2012-8-13 01:39
逻辑小分队  8月12日  【1-13】
1.    high school students who feel that they are not succeeding in high school often drop out before graduating and go to work.
Premise: last year, the dropout rate is lower than before.
Conclusion: the program began 2 years ago works.
Prophase: others reasons caused rate down
Answer: A

2.    remise: neighboring country Y has experienced the same climatic condition as Country X.  While the production rises.
Conclusion: an overly centralized, not the changes in climate, is responsible for the poor production.
Prophase: plants in two country are different.
Answer: D

3.    没看懂。
Answer: D

4.    in previous 3 years. Pharmaceuticals accounted for 20 percent of dollar sales and 40 percent of profits. And the chemicals division for the balance.
Answer: C

5.    the technological conservatism of bicycle manufactures is a reflection of the kind of demand they are trying to meet.
Premise: The only cyclists seriously interested in innovation and willing to pay for it are bicycle racers.
Conclusion: innovation in bicycle is limited by what authority would accept as standard for purposes of competition in bicycle races.
Answer: C
作者: wanggang0411    时间: 2012-8-21 16:45
1 32
Background: Most students who feel that they can not succeed in school will drop out before graduating and go to work;
Premise: The drop out rate is lower significantly than previous year.
conclusion: The program instituted to improve the morale of students take effect to reduce dropouts.
Weaken: How about there are some other factors caused this effect?

2 35
Conclusion: Centralized economy, not the changes in climates, caused the poor agricultural production X.
Premise: Country Y experienced as same climates changes as Country Y, but its agricultural rasied.
Weaken: To weaken this argument, Any answer choice points out that there are other factors caused the agriculural raising in country Y but not in country Y is a correct answer.
Answer D: The crops grown in country X is different with the crops grown in country Y.

3 more than 3 minutes
Premise: Scientists accept those with the goal of doing important research as their colleagues.
Conlucsion: Scientists who win renown as an expounder of science to general audiences will not be accpeted as true colleages by other scientists.
Inference: The scientists assume that the motivation of those popularizer become different.
Answer D

4 33
Fact sets: Pharmaceuticals division has accounted for roughly 20% of dollar sales and 40% of profits.
Fact sets: The chemical division for the balance.
Answer C: Chemical division accounted for 80% of dollar sales and 60% of profits.

5 37

Premise: The only people who insterests in innovation of bicycle is bicycle racer.
Premise: The conservatism of innovation reflects the demand that need to be met.
Conclusion: The innovation of bicycle technology is limited by what authorities will accpet as standard of bicycle races.
Assumption: The innovation will only be really required by bicycle racers when the autorities accpet it as the standard of bicycle races.
Answer C
作者: chengzaaaa    时间: 2012-8-30 21:44
2.
1)28”weaken
2)BI: if high school kid are feeling that they are not succeeding high school, they would drop out before graduating.
P: last year, the dropout rate was lower than the previous year
C: the program that was aimed to improve morale is taking effect to reduce dropout rate.
3)this year’s students are smarter or of higher quality
4)A
5)A. correct. Kids who want to drop out could not find a job, so they stayed.
B. no direct linking between morale and drop out rate
C. irrelevant comparison
D. helping to get a job is the opposite of the program
E. sort of supporting.

49.
1)17”weaken
2)BI:
P: Y experienced the same climate as X does, and the production is rising.
C: the economy is the cause of the poor agricultural production in X, not the climate.
3)other factors caused Y’s rising production
D

50.
1)33”assumption
2)scientists all want to do important new research and accept as their coworkers those with similar motivation.
When a scientist win a reward, others no longer see him/her as a true colleague.
3)when they win, they are no longer motivated to do new research
D

51.
1)19”infer
2)2 divisions. Past 3 years.
P 20%(sales) 40%(profits)
C 80%(sales) 60%(profits)
3)
C

52.
1)23”assumption
2)only cyclists seriously interested in innovation and willing to pay for it are racers.
?innovation of bikes is limited by competition purposes.
3)
C



谢谢lz!!!
作者: 我心匪席    时间: 2012-9-7 15:34
Time: 27”
Background Information: Not succeeding in high school, students will drop out the school.
Premise: Last year, the dropout rate was lower than that of previous year.
Conclusion: Programs improve the moral of high school.
Prephase: Students feel more interested in high school.
A-----right weaken
B-----irrelevant
C-----irrelevant
D-----irrelevant
E-----irrelevant
================================================

Time: 23”
Premise: Y has similar climatic conditions, the agricultural production has been falling in X, it has been rising in Y.
Conclusion: Centralized economy, not the climate, caused poor agricultural production in X.
Prephase: The circumstance for agricultural production growing is different.
A-----irrelevant
B-----right weaken
C-----irrelevant
D-----irrelevant (right)
E-----irrelevant
过度推理了
=================================================

Time: 30”
Background Information: Scientists with goal of doing important new research can be accepted by their colleagues with similar motivation.
Premise: Scientists win renown.
Conclusion: Such scientists cannot be regarded as a true colleague.
Prephase: Such scientists cannot do important new research work.
A-----irrelevant
B-----irrelevant
C-----irrelevant
D-----right
E-----irrelevant
==============================================

Time: 21”
Background Information: Pharmaceuticals division has accounted for roughly 20 percent of dollar sales and 40 percent of profits, and the chemicals division for the balance.
A-----irrelevant
B-----irrelevant
C-----right
D-----irrelevant
E-----irrelevant
===============================================

Time: 26”
Background Information: The technological conservatism of bicycle manufacturers is a reflection of the kinds of demand they are trying to meet.
Premise: Bicycle racers are interested in innovation and will pay for it.
Conclusion: innovation in bicycle technology is limited by what authorities will accept as standard for purposes of competition in bicycle races.
Prephase: Such scientists cannot do important new research work.
A-----irrelevant
B-----irrelevant
C-----right
D-----irrelevant
E-----irrelevant
作者: ElenW    时间: 2012-10-14 01:19
精练(22'9)
Bg:High school students often drop out before graduating and go to work.But last year the rate was decreased.
pre:the program instituted two years ago to improve the morale of high school
students
con:The program has begun to take effect to reduce dropouts.
(weaken)
Pre:The conclusion should be attacked.Because it not complete.There are some another reasons to lead this result.
Ans:A
B-I think it is a shell game answer
C-no impact
D-no impact
E-not complete
逻辑链49(30'4)
Bg:An overly centralized economy is responsible for the poor agricultural production in Country X since its new government came to power.
pre: Neighboring Country Y has experienced the same climatic conditions
con:agricultural production has been falling in Country X,bening rising in country Y
(weaken)
Pre:Although these countries have experienced the same climatic conditions, the agricultural production has different typies
Ans
A-not the issue
B-no impact
C-the simulu has raised that
E-irrelveant
50(21'13)
Bg:Generally scientists enter their field with the goal of doing important new research and accept as their colleagues those with similar motivation.
pre:when any scientist wins renown as an expounder of science to general audiences
con:most other scientists conclude that this popularizer should no longer be regarded as a true colleague.
(support)
Pre:the research is the subject,and they worry about the scientist who wins renown would be lost in the future.
Ans
A-no impact
B-not the issue and it vilify the scientists,
C-itself is not correct
E-irrelveant
作者: srafcatt    时间: 2012-10-19 09:12
精炼题
30s weaken
high school stu high dropout rate--this yr the rate lower than the last yr--the reason:two yr program
weaken by menly lower than last yr,so what?
choice:C
(A) There was a recession that caused a high level
of unemployment in the city.  irrelevant
(B) The morale of students who dropped out of
high school had been low even before they
reached high school.  strenghten
(C) As in the preceding year, more high school
students remained in school than dropped
out.  irrelevant
(D) High schools in the city established placement
offices to assist their graduates in obtaining
employment.   correct,it offers another explanation.
(E) The antidropout program was primarily aimed
at improving students’ morale in those high
school with the highest dropout rates.  strenghten
逻辑链
1. 26s  weaken
overly centralized eco--poor agri production in X
weaken by me:Y is centralized, too
choice
(A) Industrial production also is declining in Country X. irrelevant
(B) Whereas Country Y is landlocked, Country X has a major seaport.  irrelevant
(C) Both Country X and Country Y have been experiencing drought conditions.  strenghten
(D) The crops that have always been grown in Country X are different from those that have always been grown in Country Y.  correct
(E) Country X's new government instituted a centralized economy with the intention of ensuring an equitable distribution of goods  irrelevant

2,39s  assumption
scientist regard people with same motivation as collegue--when they win renown they no longer do so.
my assumption: those won renown will no longer willing to do research.
choice
(A) serious scientific research is not a solitary activity, but relies on active cooperation among a group of colleagues irrelevant
(B) research scientists tend not to regard as colleagues those scientists whose renown they envy irrelevant
(C) a scientist can become a famous popularizer without having completed any important research ridiculous
(D) research scientists believe that those who are well known as popularizers of science are not motivated to do important new research  correct
(E) no important new research can be accessible to or accurately assessed by those who are not themselves scientists  nonsense
3. 29s inference
premise:2 division--pharmacenticals 20 sales & 40 profits
                    chemical for the balance
my inference: may be it is time to enlarge the sales and profits by expanding the pharmacenticals department.
choice:C
(A) Total dollar sales for each of the company's divisions have remained roughly constant.    what is dollar sales?
(B) The pharmaceuticals division has faced stiffer competition in its markets than has the chemicals division. not convincing
(C) The chemicals division has realized lower profits per dollar of sales than has the pharmaceuticals division.   correct
(D) The product mix offered by each of the company's divisions has remained unchanged.  who knows?it isn't mentioned.
(E) Highly profitable products accounted for a higher percentage of the chemicals division's sales than of those of the pharmaceuticals division.  not mentioned.

4.  25s  assumption
innovation paid by races,while normal bike paid by normal people.
my assumption: the way to support innovation can only come from sales.
choice:C
(A) The market for cheap, traditional bicycles cannot expand unless the market for high-performance competition bicycles expands. irrelevant
(B) High-performance bicycles are likely to be improved more as a result of technological innovations developed in small workshops than as a result of technological innovations developed in major manufacturing concerns.     the workshop is not mentioned
(C) Bicycle racers do not generate a strong demand for innovations that fall outside what is officially recognized as standard for purposes of competition.  CORRECT
(D) The technological conservatism of bicycle manufacturers results primarily from their desire to manufacture a product that can be sold without being altered to suit different national markets. this is the opposite of the fact listed in the question stem.
(E) The authorities who set standards for high-performance bicycle racing do not keep informed about innovative bicycle design. same reason as the choice D
作者: srafcatt    时间: 2012-10-19 09:46
纠错:精炼题
再次因为 偷换对象选错答案。 这里面有一个规律:首先要找到题干在谈论的中心。本题是high school dropout。然后是找逻辑关系:本题是一个两年的鼓舞士气的项目使high school dropout 下降的。


(A) There was a recession that caused a high level
of unemployment in the city.  
分析:
这里的recession 和unemployment 联系起来了,是因果关系,题干中提到了辍学的学生的去向——go to work所以本选项的逻辑链补充在:recession--unemployment--无法go to work--dropout rate is lower.



(D) High schools in the city established placement
offices to assist their graduates in obtaining
employment.  
分析:
偷换对象的选项会造成逻辑链断裂:
本项逻辑链是:placement office--graduates employment    so what?那些称得上要辍学的还未毕业,轮不上呀。想工作,辍学好了,用你帮忙找工作。
因此,骗局(shell game)常常给出相似的概念,如果本项是学校帮助所有学生找工作,只要他们不辍学,还好一点,只可惜,听起来也没有A项那么有说服力。因为可以有人说,我现在就工作,不用你找。而A是自己找不到工作,不得不留下来了。
作者: hanhan1991    时间: 2012-11-24 11:54
1.weaken
1)time:28'/1''06'
2)logical lines:
students who feel not succeeding in school often dropout and go to work.
last year's dropout rate declined->the program of improving students moral works.
3)Prediction:
There must be some reason other than implementing the program that leads the reduced dropout rate. For example, because of the unfavorable economic situation, student may found hard to find a job last year, so they stayed at school.
4)Answer:A
A.the same as prediction
B.slighly strengthen the argument.
C.irrelevant.
D.irrelevant.
E.irrelevant.

2.weaken
1)time:24'/50'
2)logical lines:
agricultural corps grew well in country Y but not in country X though these two countries experienced the same climate condition last year.->it is the centralized economy but not the climate caused the low producation of corps in country X.
3)Prediction:
In order to weaken the argument, we need to prove that it is the climate that affacted the growth of the corps in country X. Possibly, the kinds of corps cultivated in X and Y are different, so these two kinds of corps may adjust to different climate.
4)answer

3.assumption
1)time:35'/1''06'
2)logical lines:
scientist consider themselves and their colleagues with the goal of doing important new research.->?->any scientist renow as an expounder of science to general public are not the real scientist.
3)prediction:
There's a gap in the logical lines, so we need to fill in the gap that any scientists who renown as an expounder of science to general public are not with the goal of doing important new research.
4)answer

4.must be true
1)time:34'/1''03'
2)logical lines:
pharmaceutical divisions has accounted for 20 percent of dollar sales and 40 percent of profits.
3)Prediction:
the chemical division accounted for 80 percent of dollar sales and 60 percent of profits, so the profits per dollar are higher for pharmacetical division.
4)answer:C

5.assumption
1)time:32'/1''16'
2)logical lines:
innovation reflect people's buying tendency & the main buyer are cycle racers->?->the innovations are limitied to the standards of competition race.
3)Predication:
there's a gap in the reasoning, so we need to fill in it. cycle racers' desire to buy the new innovation are limited by the standards of competition race.
4)answer:C
作者: zxppx    时间: 2013-1-14 19:43
2.

High school students who feel that they are not
succeeding in high school often drop out before
graduating and go to work. Last year, however, the
city’s high school dropout rate was significantly
lower than the previous year’s rate. This is
encouraging evidence that the program instituted
two years ago to improve the morale of high school
students has begun to take effect to reduce dropouts.

The argument attributes the lower high school dropout to the program instituted two years ago to improve the morale of high school students. In order to weaken the reasoning, we need to find other evidence that contributed to the lower dropout.
Which one of the following, if true about the last
year, most seriously weakens the argument?
(A) There was a recession that caused a high level
of unemployment in the city.

Since the recession is the reasonable factor (alternative cause) that can contribute the lower dropout, the argument can be weakened. If the job market need not so many people, those students willing to go to work would have to stay in high school. BA
(B) The morale of students who dropped out of
high school had been low even before they
reached high school.

It cannot weaken the argument, because when they enter high school, those students’ morale can still be improved by the program.
(C) As in the preceding year, more high school
students remained in school than dropped
out.

It cannot weaken the argument.
(D) High schools in the city established placement
offices to assist their graduates in obtaining
employment.

In this case, it actually strengthens the argument. Because even though it is easier for those students to go to work, the dropout rate still decreases.

Besides, this is a Shell Game answer. It aims at the high school graduate rate, rather than dropout rate.
(E) The antidropout program was primarily aimed
at improving students’ morale in those high
school with the highest dropout rates.

It cannot weaken the argument.



49.
An overly centralized economy, not the changes in the climate, is responsible for the poor agricultural production in Country X since its new government came to power. Neighboring Country Y has experienced the same climatic conditions, but while agricultural production has been falling in Country X, it has been rising in Country Y.
Which of the following, if true, would most weaken the argument above?

(A) Industrial production also is declining in Country X.

It seems to strengthen the argument, rather than weaken.
(B) Whereas Country Y is landlocked, Country X has a major seaport.

It seems to be a contender, but the argument concerns the agricultural productions between X and Y, rather the locations between them.
(C) Both Country X and Country Y have been experiencing drought conditions.

The same to A.
(D) The crops that have always been grown in Country X are different from those that have always been grown in Country Y.

Yes! If the crops grown between the two countries are different, even though the climates in these two countries are same, the agricultural productions between them may still be different, thus having no relationship with the overly centralized economy. BA
(E) Country X's new government instituted a centralized economy with the intention of ensuring an equitable distribution of goods.

It is outside the scope of the argument.



50.
Generally scientists enter their field with the goal of doing important new research and accept as their colleagues those with similar motivation. Therefore, when any scientist wins renown as an expounder of science to general audiences, most other scientists conclude that this popularizer should no longer be regarded as a true colleague.
The explanation offered above for the low esteem in which scientific popularizers are held by research scientists assumes that

A) serious scientific research is not a solitary activity, but relies on active cooperation among a group of colleagues

Shell Game: the argument concerns about generally scientists, rather than serious scientific research.
(B) research scientists tend not to regard as colleagues those scientists whose renown they envy

The argument has no relationship the ‘scientists whose renown are envied’.
(C) a scientist can become a famous popularizer without having completed any important research

No evidence is provided by the argument about this.
(D) research scientists believe that those who are well known as popularizers of science are not motivated to do important new research

The gap of the argument is why renowned scientists are not deemed to be the true colleagues by most other scientists. Amazingly, the answer choice points out the gap and makes the logic perfect, because it reasons that those famous scientists may not do important new research referred in the first sentence of the argument. BA
(E) no important new research can be accessible to or accurately assessed by those who are not themselves scientists

Those who are not scientists are out of the scope of the argument.



51.
A company's two divisions performed with remarkable consistency over the past three years: in each of those years, the pharmaceuticals division has accounted for roughly 20 percent of dollar sales and 40 percent of profits, and the chemicals division for the balance.
Which of the following can properly be inferred regarding the past three years from the statement above?

(A) Total dollar sales for each of the company's divisions have remained roughly constant.

Actually, we know nothing about total dollar sales for each of the divisions.
(B) The pharmaceuticals division has faced stiffer competition in its markets than has the chemicals division.


It is the opposite answer.
(C) The chemicals division has realized lower profits per dollar of sales than has the pharmaceuticals division.

Yes, we can see that profits per dollar of sales are higher in the pharmaceuticals than in the chemicals division. BA
(D) The product mix offered by each of the company's divisions has remained unchanged.

Actually, we do not know the product mix of each of the company’s divisions.
(E) Highly profitable products accounted for a higher percentage of the chemicals division's sales than of those of the pharmaceuticals division.

In fact, we know nothing about the sales construction between these two divisions.



52.
The technological conservatism of bicycle manufacturers is a reflection of the kinds of demand they are trying to meet. The only cyclists seriously interested in innovation and willing to pay for it are bicycle racers. Therefore, innovation in bicycle technology is limited by what authorities will accept as standard for purposes of competition in bicycle races.
Which of the following is an assumption made in drawing the conclusion above?

(A) The market for cheap, traditional bicycles cannot expand unless the market for high-performance competition bicycles expands.

The market for cheap, traditional bicycles is outside the scope of the argument.
(B) High-performance bicycles are likely to be improved more as a result of technological innovations developed in small workshops than as a result of technological innovations developed in major manufacturing concerns.

The comparison between small workshops and major manufacturing concerns is irrelevant.
(C) Bicycle racers do not generate a strong demand for innovations that fall outside what is officially recognized as standard for purposes of competition.

The gap of the argument is the relationship between bicycle racers and what authorities will accept as standard for purposes of competition in bicycle races. Amazingly, this answer choice finds out the gap and makes the logic reasonable. BA
(D) The technological conservatism of bicycle manufacturers results primarily from their desire to manufacture a product that can be sold without being altered to suit different national markets.

The reason of the technological conservatism of bicycle manufacturers is irrelevant.
(E) The authorities who set standards for high-performance bicycle racing do not keep informed about innovative bicycle design.

Actually, it weakens the argument in some degree.

作者: CD用户825193    时间: 2013-1-31 09:28
标题: Daily CR-13_2013-01-31 / 02-01
2. High school students who feel that they are not
succeeding in high school often drop out before
graduating and go to work. Last year, however, the
city’s high school dropout rate was significantly
lower than the previous year’s rate. This is
encouraging evidence that the program instituted
two years ago to improve the morale of high school
students has begun to take effect to reduce dropouts.
Which one of the following, if true about the last
year, most seriously weakens the argument?
----------Cause and Effect Problems NO.2

30" (Weak)
P: Students not succeeding in high school normally drop out.
P: Last yr dropout rate is lower than previous.P: 2 yrs ago started a program to lower the dropout rate.
C: the program caused the lower rate.
Pre: More students who have high possibility to drop out didn't register to high school.
   some other reasons actually caused the lower rate

Answers:
(A) There was a recession that caused a high level
of unemployment in the city.

   unemployment doesn't necessarily stop students dropping out.
succeeding: won't go to college
CORRECT: not succeeding & get employment -> dropouts

(B) The morale of students who dropped out of
high school had been low even before they
reached high school.
   before hight school is out of the topic

(C) As in the preceding year, more high school
students remained in school than dropped
out.
   support  irrelevant

(D) High schools in the city established placement
offices to assist their graduates in obtaining
employment.
   CORRECT, this could keep students in school

(E) The antidropout program was primarily aimed
at improving students’ morale in those high
school with the highest dropout rates.
   support  "aimed at overall high schools" is only a description of the program, not helpful to explain or weak the argument

49.
An overly centralized economy, not the changes in the climate, is responsible for the poor agricultural production in Country X since its new government came to power.  Neighboring Country Y has experienced the same climatic conditions, but while agricultural production has been falling in Country X, it has been rising in Country Y.
Which of the following, if true, would most weaken the argument above?

25" (Weak)
P: Agricultural production is dropping in Country X, while Country Y has increasing agricultural production
P: Climate changes and new government in X
C: It's the economic change rather than climate change influenced the agricultural drop in X
Pre: X & Y have different types of agricultural plants/soil/ect.
    X's new government encourages agriculture somewhat

Answers:
(A) Industrial production also is declining in Country X.
    not related

(B) Whereas Country Y is landlocked, Country X has a major seaport.

    irrelevant to agricultural production

(C) Both Country X and Country Y have been experiencing drought conditions.

    it's climate change, didn't weaken

(D) The crops that have always been grown in Country X are different from those that have always been grown in Country Y.

    CORRECT.

(E) Country X's new government instituted a centralized economy with the intention of ensuring an equitable distribution of goods.

    new economy policy, but doesn't weaken
50.
Generally scientists enter their field with the goal of doing important new research and accept as their colleagues those with similar motivation.  Therefore, when any scientist wins renown as an expounder of science to general audiences, most other scientists conclude that this popularizer should no longer be regarded as a true colleague.
The explanation offered above for the low esteem in which scientific popularizers are held by research scientists assumes that


1'03" (presume)
P: Scientists with the goal of doing important research and same motivation are considered colleagues by others
Pre: those who got popularized don't do important research or with the same motivation as others
C: Scientists who won the renown or get popular no longer being considered others' colleagues

Answers:
(A) serious scientific research is not a solitary activity, but relies on active cooperation among a group of colleagues
    nothing related with popularizer

(B) research scientists tend not to regard as colleagues those scientists whose renown they envy
    envy is out of the topic

(C) a scientist can become a famous popularizer without having completed any important research
    it's only one possibility

(D) research scientists believe that those who are well known as popularizers of science are not motivated to do important new research
    CORRECT

(E) no important new research can be accessible to or accurately assessed by those who are not themselves scientists
    -> important new research can only be accessible to or ... by scientists
51.
A company's two divisions performed with remarkable consistency over the past three years:  in each of those years, the pharmaceuticals division has accounted for roughly 20 percent of dollar sales and 40 percent of profits, and the chemicals division for the balance.

Which of the following can properly be inferred regarding the past three years from the statement above?


33" (Conclusion)
P: 2 divisions of a company: pharmaceuticals and chemicals r doing good
P: P's sales and profits increased 20% and 40% respectively, C keeps balance
C: P's cost decreases more than C's or P's price increases more than C's

Answers:
(A) Total dollar sales for each of the company's divisions have remained roughly constant.
    can't tell the exact dollar sales

(B) The pharmaceuticals division has faced stiffer competition in its markets than has the chemicals division.
    shall be the opposite

(C) The chemicals division has realized lower profits per dollar of sales than has the pharmaceuticals division.
    CORRECT

(D) The product mix offered by each of the company's divisions has remained unchanged.
    can't tell about the change

(E) Highly profitable products accounted for a higher percentage of the chemicals division's sales than of those of the pharmaceuticals division.
    shall be opposite
52.
The technological conservatism of bicycle manufacturers is a reflection of the kinds of demand they are trying to meet.  The only cyclists seriously interested in innovation and willing to pay for it are bicycle racers.  Therefore, innovation in bicycle technology is limited by what authorities will accept as standard for purposes of competition in bicycle races.
Which of the following is an assumption made in drawing the conclusion above?


25" (Missing P)
P:
P:
C: authorities of bicycle races influence -> cyclists demands -> manufacturers technical change
Pre:

Answers:
(A) The market for cheap, traditional bicycles cannot expand unless the market for high-performance competition bicycles expands.
    cheap bicycles r not related

(B) High-performance bicycles are likely to be improved more as a result of technological innovations developed in small workshops than as a result of technological innovations developed in major manufacturing concerns.
    where bicycles get developed is not related

(C) Bicycle racers do not generate a strong demand for innovations that fall outside what is officially recognized as standard for purposes of competition.
    CORRECT

(D) The technological conservatism of bicycle manufacturers results primarily from their desire to manufacture a product that can be sold without being altered to suit different national markets.
    It's about the races/authorities, not different national markets

(E) The authorities who set standards for high-performance bicycle racing do not keep informed about innovative bicycle design.
    it's the other way round, but not a necessary pre to the conclusion
作者: pennyz    时间: 2013-2-15 22:55
1:47
weaken(last year)
method: program to increase high school morality
result:less dropout last year
weaken: other reason for less dropout
a----correct


(B) The morale of students who dropped out of
high school had been low even before they
reached high school.------irrelevant
(C) As in the preceding year, more high school
students remained in school than dropped
out.----support
(D) High schools in the city established placement
offices to assist their graduates in obtaining
employment.-----irrelevant, no info about office
(E) The antidropout program was primarily aimed
at improving students’ morale in those high
school with the highest dropout rates.
-----irrelevant
作者: pennyz    时间: 2013-2-15 23:30
1:12s
weaken
d
cause-result
centralize e--->argi fall X
y rise
give other differences
1:49s
assumption
c
premise:scientist do imp new research/ accept  similar motivation colleagues
conclusion:popularity---->no true collegue
1:43
two division is constant in 3 year
phram:20% sale 40% profit
chem:meet the end
c
1:59s
premise: bicycle is only interested by competitors
conclusion:allow innovation within the compition use
c
作者: Feelalive    时间: 2013-7-12 14:44
Feelalive到此一游
作者: Extremo    时间: 2013-7-19 16:58
挖坟补作业咯
作者: kevinlovett    时间: 2013-7-20 17:05
fdghbghsrgsrg
作者: 灵药要奋斗    时间: 2013-8-21 19:09
这不都是OG上做过的题目么。。。
作者: Elisha728    时间: 2013-8-31 19:20
5'44''
ADDCC
作者: Mint静默    时间: 2013-9-9 15:22
1) 计时:26s
2) 逻辑链
Background Information:
High school students who feel that they are not succeeding in high school often drop out before graduating and go to work.
Premise:
Last year, however, the city’s high school dropout rate was significantly lower than the previous year’s rate.
Conclusion:
The program instituted two years ago to improve the morale of high school students has begun to take effect to reduce dropouts.
3) 推测:(weaken)
dropout rate 下降有它因,比如说学生总数上升
4) 选项分析:
(A) There was a recession that caused a high level of unemployment in the city. Correct
(B) The morale of students who dropped out of high school had been low even before they reached high school. Irrelevant
(C) As in the preceding year, more high school students remained in school than dropped out. Irrelevant
(D) High schools in the city established placement offices to assist their graduates in obtaining employment. 反,这样的话学生就会更加倾向于提前找工作
(E) The antidropout program was primarily aimed at improving students’ morale in those high schools with the highest dropout rates. Irrelevant

1) 计时:20s
2) 逻辑链
Premise:
Neighboring Country Y has experienced the same climatic conditions, but while agricultural production has been falling in Country X, it has been rising in Country Y.
Conclusion:
An overly centralized economy, not the changes in the climate, is responsible for the poor agricultural production in Country X since its new government came to power.
3) 推测:(weaken)
这个argument只能一定程度上否定climate不是原因,而不能支持overly centralized economy是原因。有他因
选D

1) 计时:44s
2)
逻辑链

Premise:
Generally scientists enter their field with the goal of doing important new research and accept as their colleagues those with similar motivation.
Conclusion:
When any scientist wins renown as an expounder of science to general audiences, most other scientists conclude that this popularizer should no longer be regarded as a true colleague.
3)
推测:(assumption
Popularizer does not share similar motivation (doing new researches) with general scientists
D
1) 计时:18s
2)
逻辑链
Background Information:
A company's two divisions performed with remarkable consistency over the past three years.
Premise:
In each of those years, the pharmaceuticals division has accounted for roughly 20 percent of dollar sales and 40 percent of profits, and the chemicals division for the balance.
3) 推测:(Infer)
注意理解balance是什么意思,balance的意思是两个部门的profit percentagesales percentage分别相加等于一
C
1) 计时:80s (看了好久0 0
2) 逻辑链
Premise:
Only bicycle racers are seriously interested in innovation and willing to pay for it.
Conclusion:
Innovation in bicycle technology is limited by what authorities will accept as standard for purposes of competition in bicycle races.
3) 推测:(Assumption)
Bicycle racers buy the innovated bicycles only for bicycle races.
C


作者: lyrsilvia    时间: 2013-9-18 19:13
A DDCC
1.18'
BG: not succeed in high school-->drop out before graduate and go to work
P: last year high school dropout rate lower
C: the program instituted 2 years ago to improve the morale of high school students has begun to take effect to reduce dropouts
Weaken: the lower dropout rate has nothing to do with the instituted program
答案:A
(A) There was a recession that caused a high level of unemployment in the city.------------CORRECT. means that it is not the program but the high unemployment rate that keeps students in school.
(B) The morale of students who dropped out of high school had been low even before they reached high school.-----------whether the morale is lower before high school does not affect the success of the program
(C) As in the preceding year, more high school students remained in school than dropped out.-----------the similarity shows the instituted program is successful
(D) High schools in the city established placement offices to assist their graduates in obtaining employment.-------------whether assist graduates in obtaining employment does not reflect the program's success or failure
(E) The anti-dropout program was primarily aimed at improving students’ morale in those high school with the highest dropout rates.-------------where the program is aimed or instruct does not reflect the effectiveness of the program.

2. 20'
P: climate change, but the agricultural production in Y rise while production in X reduce
C: the bad situation is due to the overly centralized economy
Weaken: the crops in X and in Y are different, climate change has different influence for them
答案:D
(A) Industrial production also is declining in Country X.---------another adverse evidence to support the bad government.
(B) Whereas Country Y is landlocked, Country X has a major seaport.--------this is irrelevant. no useful information is provided to judge
(C) Both Country X and Country Y have been experiencing drought conditions.----------the repetition of the PREMISE, meaningless.
(D) The crops that have always been grown in Country X are different from those that have always been grown in Country Y.-----------CORRECT. maybe crops in Y are more strong to the bad climate change while crops in X are more fragile.
(E) Country X's new government instituted a centralized economy with the intention of ensuring an equitable distribution of goods.-----------confer the overly centralized economy situation

3. 20'
BG: enter the science field with the goal of doing important new research & accept colleagues with similar motivation
P: scientist wins renown as an expounder of science to general audiences
C: most other scientist conclude that this popularizer should no longer be regarded as a true colleague
Assumption: popularizer means other motivation
答案:D
(A) serious scientific research is not a solitary activity, but relies on active cooperation among a group of colleagues-------------irrelevant information about serious scientific research
(B) research scientists tend not to regard as colleagues those scientists whose renown they envy----------"the renown is envy by scientists" this statement has no other information to be supported
(C) a scientist can become a famous popularizer without having completed any important research-------------how a scientist becomes famous does not reflect the reason why he is not regarded as colleagues
(D) research scientists believe that those who are well known as popularizers of science are not motivated to do important new research-------------CORRECT. the premise said "accept as their colleagues those with similar motivation".thus the reason why popularizers are not colleagues must related to this.
(E) no important new research can be accessible to or accurately assessed by those who are not themselves scientists ------------what can be accessible is not discussed in this question. compare with D, you will know the reasoning.

4. 16'
P: pharmaceuticals: 20% sales & 40% profits
    chemicals: 80% sales & 60% profits
CONCLUSION: pharmaceuticals more profitable
答案:C
(A) Total dollar sales for each of the company's divisions have remained roughly constant.-------performed consistency≠total dollar sales remained constant. 比例不变≠总量不变
(B) The pharmaceuticals division has faced stiffer competition in its markets than has the chemicals division.--------------stiffer competition should lead to less profits, not more profitable situation
(C) The chemicals division has realized lower profits per dollar of sales than has the pharmaceuticals division.---------------CORRECT. simple percentage comparison
(D) The product mix offered by each of the company's divisions has remained unchanged.------------there is no information about the product mix. IRRELEVANT
(E) Highly profitable products accounted for a higher percentage of the chemicals division's sales than of those of the pharmaceuticals division.-------------if highly profitable products accounted for higher percentage of the chemicals division's sales, the sales in chemical division should over 80% but not just 60%

5. 24'
P: The only cyclists seriously interested in innovati
作者: lyrsilvia    时间: 2013-9-18 19:15
A DDCC
1.18'
BG: not succeed in high school-->drop out before graduate and go to work
P: last year high school dropout rate lower
C: the program instituted 2 years ago to improve the morale of high school students has begun to take effect to reduce dropouts
Weaken: the lower dropout rate has nothing to do with the instituted program
答案:A
(A) There was a recession that caused a high level of unemployment in the city.------------CORRECT. means that it is not the program but the high unemployment rate that keeps students in school.
(B) The morale of students who dropped out of high school had been low even before they reached high school.-----------whether the morale is lower before high school does not affect the success of the program
(C) As in the preceding year, more high school students remained in school than dropped out.-----------the similarity shows the instituted program is successful
(D) High schools in the city established placement offices to assist their graduates in obtaining employment.-------------whether assist graduates in obtaining employment does not reflect the program's success or failure
(E) The anti-dropout program was primarily aimed at improving students’ morale in those high school with the highest dropout rates.-------------where the program is aimed or instruct does not reflect the effectiveness of the program.

2. 20'
P: climate change, but the agricultural production in Y rise while production in X reduce
C: the bad situation is due to the overly centralized economy
Weaken: the crops in X and in Y are different, climate change has different influence for them
答案:D
(A) Industrial production also is declining in Country X.---------another adverse evidence to support the bad government.
(B) Whereas Country Y is landlocked, Country X has a major seaport.--------this is irrelevant. no useful information is provided to judge
(C) Both Country X and Country Y have been experiencing drought conditions.----------the repetition of the PREMISE, meaningless.
(D) The crops that have always been grown in Country X are different from those that have always been grown in Country Y.-----------CORRECT. maybe crops in Y are more strong to the bad climate change while crops in X are more fragile.
(E) Country X's new government instituted a centralized economy with the intention of ensuring an equitable distribution of goods.-----------confer the overly centralized economy situation

3. 20'
BG: enter the science field with the goal of doing important new research & accept colleagues with similar motivation
P: scientist wins renown as an expounder of science to general audiences
C: most other scientist conclude that this popularizer should no longer be regarded as a true colleague
Assumption: popularizer means other motivation
答案:D
(A) serious scientific research is not a solitary activity, but relies on active cooperation among a group of colleagues-------------irrelevant information about serious scientific research
(B) research scientists tend not to regard as colleagues those scientists whose renown they envy----------"the renown is envy by scientists" this statement has no other information to be supported
(C) a scientist can become a famous popularizer without having completed any important research-------------how a scientist becomes famous does not reflect the reason why he is not regarded as colleagues
(D) research scientists believe that those who are well known as popularizers of science are not motivated to do important new research-------------CORRECT. the premise said "accept as their colleagues those with similar motivation".thus the reason why popularizers are not colleagues must related to this.
(E) no important new research can be accessible to or accurately assessed by those who are not themselves scientists ------------what can be accessible is not discussed in this question. compare with D, you will know the reasoning.

4. 16'
P: pharmaceuticals: 20% sales & 40% profits
    chemicals: 80% sales & 60% profits
CONCLUSION: pharmaceuticals more profitable
答案:C
(A) Total dollar sales for each of the company's divisions have remained roughly constant.-------performed consistency≠total dollar sales remained constant. 比例不变≠总量不变
(B) The pharmaceuticals division has faced stiffer competition in its markets than has the chemicals division.--------------stiffer competition should lead to less profits, not more profitable situation
(C) The chemicals division has realized lower profits per dollar of sales than has the pharmaceuticals division.---------------CORRECT. simple percentage comparison
(D) The product mix offered by each of the company's divisions has remained unchanged.------------there is no information about the product mix. IRRELEVANT
(E) Highly profitable products accounted for a higher percentage of the chemicals division's sales than of those of the pharmaceuticals division.-------------if highly profitable products accounted for higher percentage of the chemicals division's sales, the sales in chemical division should over 80% but not just 60%

5. 24'
P: The only cyclists seriously interested in innovation and willing to pay for it are bicycle racers.
C: innovation in bicycle technology is limited by what authorities will accept as standard for purposes of competition in bicycle races.
Assumption: bicycle racers interests=authorities accept
答案:C
(A) The market for cheap, traditional bicycles cannot expand unless the market for high-performance competition bicycles expands.-------------------if market for cheap, traditional bicycle can expand, there still no guarantee about the innovation
(B) High-performance bicycles are likely to be improved more as a result of technological innovations developed in small workshops than as a result of technological innovations developed in major manufacturing concerns.-------------------states the advantages of innovation, but no information about the innovation guarantee
(C) Bicycle racers do not generate a strong demand for innovations that fall outside what is officially recognized as standard for purposes of competition.--------------------CORRECT. means racers=standard, thus no innovation. if not, there still a demand for innovation
(D) The technological conservatism of bicycle manufacturers results primarily from their desire to manufacture a product that can be sold without being altered to suit different national markets.----------------states the reason about the conservation, but it is out of scope. 取非. if not, then noting can be concluded
(E) The authorities who set standards for high-performance bicycle racing do not keep informed about innovative bicycle design.--------------取非, if the authorities do keep informed about innovative bicycle design, then just informed cannot guarantee the demand for the innovation. if racers just want to fit in the standard, they will not care the innovative design.
作者: 唐韵    时间: 2013-9-18 20:36
because the students who drop out can't find a good job like the older student.
students feel not success-> drop out> drop out rate reduce> encourage>他因


两个相近的国家,同样的气候条件,一个国家由于过度的中央集权(削弱部分),导致农作物产量下降,而另一个国家的产量反而上升。
并不是因为中央集权
他因》》》》》土地的原因,农作物的原因。。不同于国家政治体制的原因

科学家有目标和接受同样想法的同志》》科学发言人并不被人们视为同僚》》解释为什么被同事瞧不起?
因为他们独占?综合其他人思想?不是自己最初想出来的?什么都不做就得到荣誉?
correct:前面提到了目标和想法

conservation> innovation only interested by racer>>>the innovation is limited by what authorities standard competition. ____assumption draw
innovation is not only interested by racer?some people who buy innovation not limited by standard?

two remarkable consistency, the P have the profile,but C don't>>>>why? C'market is very conpetitate? C per unit have less profit? more cost?lower price? than P?
作者: 览物之情    时间: 2013-10-15 04:27
10月14日 1-13
1 21s
Background: High school students who feel that they are not
succeeding in high school often drop out before
graduating and go to work. Last year,
Premise: the city’s high school dropout rate was significantly lower than the previous year’s rate.
Conclusion: the program instituted to improve the morale of high school
students has begun to take effect to reduce dropouts.

Which one of the following, if true about the last year, most seriously weakens the argument?
现象解释:P maybe other factor such as low employment rate account for this.
A)        There was a recession that caused a high level of unemployment in the city.----correct
(B) The morale of students who dropped out of high school had been low even before they
reached high school.------比较对象不对
(C) As in the preceding year, more high school students remained in school than dropped
out.
(D) High schools in the city established placement offices to assist their graduates in obtaining employment.
(E) The antidropout program was primarily aimedat improving students’ morale in those high school with the highest dropout rates.

2 20s
Conclusion: An overly centralized economy, not the changes in the climate, is responsible for the poor agricultural production in Country X
Premise: since its new government came to power.  Neighboring Country Y has experienced the same climatic conditions, but while agricultural production has been falling in Country X, it has been rising in Country Y.

Which of the following, if true, would most weaken the argument above?
P 类比推理weaken二者的本质不同
A)        Industrial production also is declining in Country X.----support
(B) Whereas Country Y is landlocked, Country X has a major seaport.
(C) Both Country X and Country Y have been experiencing drought conditions.---repeat background
(D) The crops that have always been grown in Country X are different from those that have always been grown in Country Y.--------correct
(E) Country X's new government instituted a centralized economy with the intention of ensuring an equitable distribution of goods.------irrelevant

3 38s
Premise: Generally scientists enter their field with the goal of doing important new research and accept as their colleagues those with similar motivation.  
Conclusion: when any scientist wins renown as an expounder of science to general audiences, most other scientists conclude that this popularizer should no longer be regarded as a true colleague.
The explanation offered above for the low esteem in which scientific popularizers are held by research scientists assumes that
P 问assumption support conclusion的。没懂啥思路,看下选项
A)        serious scientific research is not a solitary activity, but relies on active cooperation among a group of colleagues
(B) research scientists tend not to regard as colleagues those scientists whose renown they envy
(C) a scientist can become a famous popularizer without having completed any important research---irrelevant
(D) research scientists believe that those who are well known as popularizers of science are not motivated to do important new research-----correct
(E) no important new research can be accessible to or accurately assessed by those who are not themselves scientists

4 20s
A company's two divisions performed with remarkable consistency over the past three years:  in each of those years, the pharmaceuticals division has accounted for roughly 20 percent of dollar sales and 40 percent of profits, and the chemicals division for the balance.
Which of the following can properly be inferred regarding the past three years from the statement above?
演绎推理 P is more profitable than C
(A)        Total dollar sales for each of the company's divisions have remained roughly constant.――NM
(B) The pharmaceuticals division has faced stiffer competition in its markets than has the chemicals division.―――NM
(C) The chemicals division has realized lower profits per dollar of sales than has the pharmaceuticals division.――correct
(D) The product mix offered by each of the company's divisions has remained unchanged.
(E) Highly profitable products accounted for a higher percentage of the chemicals division's sales than of those of the pharmaceuticals division.___NM

5 23s
Premise1:The technological conservatism of bicycle manufacturers is a reflection of the kinds of demand they are trying to meet.  Premise2The only cyclists seriously interested in innovation and willing to pay for it are bicycle racers.
Conclusion: Therefore, innovation in bicycle technology is limited by what authorities will accept as standard for purposes of competition in bicycle races.
Which of the following is an assumption made in drawing the conclusion above?
证明是冲要条件。No other factor can lead to innovation?
(A) The market for cheap, traditional bicycles cannot expand unless the market for high-performance competition bicycles expands.
(B) High-performance bicycles are likely to be improved more as a result of technological innovations developed in small workshops than as a result of technological innovations developed in major manufacturing concerns.
(C) Bicycle racers do not generate a strong demand for innovations that fall outside what is officially recognized as standard for purposes of competition.----correct
(D) The technological conservatism of bicycle manufacturers results primarily from their desire to manufacture a product that can be sold without being altered to suit different national markets.
(E) The authorities who set standards for high-performance bicycle racing do not keep informed about innovative bicycle design.

作者: w.melhere    时间: 2013-10-26 12:03
感觉今天题做的不太好啊。
1. 37'
2.23' 农作物的产量和经济体制并没有很大关系。这个以前做过,今天才比较明白了。
3.43'
4.31'
5.46' 完全被E选项的各种修饰词迷惑了。果然认真我就输了啊,自己又对着E选项不停发散。并且在最后把假设题的思维和削弱搞混了。觉得31#jetyxo对这道题的逻辑很清晰呢。这道题很受用!
作者: yuehuasunday    时间: 2013-11-14 07:31
he technological conservatism of bicycle manufacturers is a reflection of the kinds of demand they are trying to meet.  
作者: yuehuasunday    时间: 2013-11-15 16:45
1.


Background:  High school students who feel that they are not
succeeding in high school often drop out before
Graduating and go to work.

Premise: Last year, the high school dropout rate was significantly lower than previous year

Conclusion: Program instituted two years ago to improve the morale of high school students has begun to take effect to reduce dropouts.

(A) Correct:  Causation

2.

Background: An overly centralized economy is responsible for the poor agricultural production.

Premise: Y experienced the same climatic

Conclusion: But Y is raising

(D) Correct

3.

Background: Generally scientists enter their field with the goal of doing important new research and accept as their colleagues those with similar motivation

Premise: when scientist wins renown as an expounder of science to general audiences

Conclusion: no longer be regarded as a true colleague.

(D)

4.

Background: A company's two divisions performed with remarkable consistency over the past three years:  

Premise: Pharmaceuticals division accounted for 20 percent of dollar sales

Conclusion: 40 Percent of profits

© Correct

5.

Background: The technological conservatism of bicycle manufacturers is a reflection of the kinds of demand they are trying to meet.

Premise: he only cyclists seriously interested in innovation and willing to pay for it are bicycle racers.  

Conclusion:  Therefore, innovation in bicycle technology is limited by what authorities will accept as standard for purposes of competition in bicycle races.

©

作者: irenetopia    时间: 2013-11-26 15:49
【精练】1:09
2. High school students who feel that they are not
succeeding in high school often drop out before
graduating and go to work. Last year, however, the
city’s high school dropout rate was significantly
lower than the previous year’s rate. This is
encouraging evidence that the program instituted
two years ago to improve the morale of high school
students has begun to take effect to reduce dropouts.
Which one of the following, if true about the last
year, most seriously weakens the argument?
----------Cause and Effect Problems NO.2
逻辑链:学生退学是因为找工作
今年退学率低了
是跟学校开始搞的一个激励项目有关
问削弱
思路:削弱就是退学率低跟项目无关,相关性不存在
前提提到了work
选项中一般都会有work

(A) There was a recession that caused a high level
of unemployment in the city.——bingo!因为经济萧条,失业率高,很多学生就找不到工作就继续上学呗,隔断相关性。
(B) The morale of students who dropped out of
high school had been low even before they
reached high school.——support
(C) As in the preceding year, more high school
students remained in school than dropped
out.——反驳前提?好吧排除
(D) High schools in the city established placement
offices to assist their graduates in obtaining
employment.——可能算是项目的一个具体实施,不相关
(E) The antidropout program was primarily aimed
at improving students’ morale in those high
school with the highest dropout rates.——同D,不相关

【逻辑链】
49.1:06
An overly centralized economy, not the changes in the climate, is responsible for the poor agricultural production in Country X since its new government came to power.  Neighboring Country Y has experienced the same climatic conditions, but while agricultural production has been falling in Country X, it has been rising in Country Y.
Which of the following, if true, would most weaken the argument above?

逻辑链:集中经济会影响X的农业(不是气候),因为它的新政府上台了。
Y的气候也没有变。
X的农业下降,Y的农业上升
问削弱:climate相同,找两个不同点
直接影响agriculture的

(A) Industrial production also is declining in Country X.——无关啊
(B) Whereas Country Y is landlocked, Country X has a major seaport.——没有直接影响
(C) Both Country X and Country Y have been experiencing drought conditions.——具体化前提,无关
(D) The crops that have always been grown in Country X are different from those that have always been grown in Country Y.——bingo!直接提到了有关于农产品的不同可能导致的不同结果
(E) Country X's new government instituted a centralized economy with the intention of ensuring an equitable distribution of goods.——无关

50.1:43
Generally scientists enter their field with the goal of doing important new research and accept as their colleagues those with similar motivation.  Therefore, when any scientist wins renown as an expounder of science to general audiences, most other scientists conclude that this popularizer should no longer be regarded as a true colleague.
The explanation offered above for the low esteem in which scientific popularizers are held by research scientists assumes that
逻辑链:一般科学家进入领域进行研究同时也会接受和他们有相同动机的同事
因此对于一般获得声誉的科学家,其他科学家认为这个人不是真正的同事
问假设
同事工作——因为有相同的动机
获得声誉的人——不是同事
有个hole,即获得声誉的人没有相同动机
在选项之中找动机和荣誉的关系

(A) serious scientific research is not a solitary activity, but relies on active cooperation among a group of colleagues——无关
(B) research scientists tend not to regard as colleagues those scientists whose renown they envy——嫉妒?好吧~~
(C) a scientist can become a famous popularizer without having completed any important research——无关
(D) research scientists believe that those who are well known as popularizers of science are not motivated to do important new research——bingo!填补了漏洞,取非削弱原题
(E) no important new research can be accessible to or accurately assessed by those who are not themselves scientists ——没看懂……无关

51.40S
A company's two divisions performed with remarkable consistency over the past three years:  in each of those years, the pharmaceuticals division has accounted for roughly 20 percent of dollar sales and 40 percent of profits, and the chemicals division for the balance.
Which of the following can properly be inferred regarding the past three years from the statement above?
逻辑链:两个部门,P部门销售20%利润40%
C部门销售80%利润60%
问总结:
根据题意,不能出现新信息

(A) Total dollar sales for each of the company's divisions have remained roughly constant.——没说,没提到
(B) The pharmaceuticals division has faced stiffer competition in its markets than has the chemicals division.——反了哎,排除
(C) The chemicals division has realized lower profits per dollar of sales than has the pharmaceuticals division.——bingo!销售额大,利润低。
(D) The product mix offered by each of the company's divisions has remained unchanged.——无关
(E) Highly profitable products accounted for a higher percentage of the chemicals division's sales than of those of the pharmaceuticals division.——利润高不等于产品利润高high profitable product新信息

52. 2:05
The technological conservatism of bicycle manufacturers is a reflection of the kinds of demand they are trying to meet.  The only cyclists seriously interested in innovation and willing to pay for it are bicycle racers.  Therefore, innovation in bicycle technology is limited by what authorities will accept as standard for purposes of competition in bicycle races.
Which of the following is an assumption made in drawing the conclusion above?
逻辑链:自行车的创新面临传统保守技术问题
唯一对这个创新感兴趣的是骑自行车的人
因此自行车的创新将会被限制于在自行车比赛中当局的标准
问假设
有个hole,自行车手和当局的关系
选项包含这个信息要注意

(A) The market for cheap, traditional bicycles cannot expand unless the market for high-performance competition bicycles expands.——cheap不cheap无关
(B) High-performance bicycles are likely to be improved more as a result of technological innovations developed in small workshops than as a result of technological innovations developed in major manufacturing concerns.——无关
(C) Bicycle racers do not generate a strong demand for innovations that fall outside what is officially recognized as standard for purposes of competition.——bingo!自行车手一般不会对创新强烈要求作为官方比赛标准。取非的话,如果是,自行车的创新将不会被限制了
(D) The technological conservatism of bicycle manufacturers results primarily from their desire to manufacture a product that can be sold without being altered to suit different national markets.——无关,不是制造商渴望
(E) The authorities who set standards for high-performance bicycle racing do not keep informed about innovative bicycle design.——不管keep不keep,都会影响
作者: goodluck22    时间: 2014-3-8 16:54
Premise: program instituted two years ago -> reduce dropouts
Prephrase: other reasons reduce the dropout not program
A.        Correct
B.        Background
C.        Background
D.        Support
E.        Just some background


Conclusion: centralized economy is responsible for poor agricultural
Premise:  Y and X has same climate condition  but production  y>x
Prephrase: other reasons cause the production difference
A.        out of scope
B.        just some background
C.        support
D.        CORRECT
E.        BACKGROUND

Background: enter field doing important research and accept their the same motivation
Conclusion: who win -> will not regard as colleague
Prephrase: will not do research
A.        background
B.        reverse
C.        irrelevant
D.        correct
E.        background

Premise: PHAR SALES 20% PROFIT 60%
                   CHEMI SALES:80%  PROFIT:40%
Prephrase: chem sales more but profit is much lower
A.        CANNOT IFER
B.        DON’T KNOW
C.        CORRECT
D.        DON’T KNOW
E.        SUBSET DON’T KNOW

Premise: cyclists interested is bicycle racer
Conclusion : innovation is limited
Prephrase:
A.        irrelevant
B.        subgr0up
C.        Correct
D.        Irrelevant
E.        irrelevant

作者: cyndichiang    时间: 2014-4-12 22:10
精炼:
1. 36‘’
premise: last year,the city’s high school dropout rate was significantly lower than the previous year’s rate
conclusion:the program instituted two years ago to improve the morale of high school students has begun to take effect to reduce dropouts
Q:weaken(what happened last year)
prephrase: high school students should study more than 2 years to graduate
选项:A 果推因
(A) There was a recession that caused a high level
of unemployment in the city.---Correct, this evidence is another factor that can cause the declined drop rate.它因
(B) The morale of students who dropped out of
high school had been low even before they
reached high school.----opposite, this statement means that the program did take into effect
(C) As in the preceding year, more high school
students remained in school than dropped
out.-----irrelevant,对比的是前年和去年的drop rate,而不是对比留校和缀学的比例
(D) High schools in the city established placement
offices to assist their graduates in obtaining
employment.-----opposite:this statement actually strengthens the argument
(E) The antidropout program was primarily aimed
at improving students’ morale in those high
school with the highest dropout rates.-----irrelevant, it has nothing to do with the argument, because we don't know whether this school has the highest dropout rate;out of scope;

逻辑链:
2. 27‘’
premise: agriculture production decreased in country X when the new government came into power.
premise: X and Y has the same climate condition.
conclusion: An overly centralized economy, not the climate, is responsible for the poor agricultural production
Q:weaken
prephrase: though the climate is the same ,if there are some other differences between the two, the economy is not the only factor.

3. 37''
premise:Generally scientists accept their colleagues those with similar motivation.
premise:any scientist wins renown as an expounder of science to general audiences
conclusion: these renowned scientists are not regarded as true colleague
Q: Assumption:
prephrase: establish relationship: renowned scientists have different motivation from general scientists

4.27''
BG:A company's two divisions performed with remarkable consistency over the past three years
premise:the pharmaceuticals division had roughly 20 percent of dollar sales and 40 percent of profits, and the chemicals division for the balance
Conclusion:???
Q:must be true
prephrase: 20:40<20:20?  40:20 >20:20?
              

5. 44''
premise: only bike racers concern about the innovation and pay for it
premise: authorities have standard for purposes of competition in bicycle races
conclusion: innovation is limited
Q:Assumption
prephrase: establish relationship: racers must obey the standard
               deny a weaken stimulus: bike racers will not use the bike in areas except race
作者: simonleo    时间: 2014-5-6 19:31
1.
Time:59s
P:students who feel that they are not succeeding in high school often drop out to work
last year, the drop out rate was significantly lower that previous year's.
C:program that improve the morale of high school students has begun to take effect to reduce dropouts.
Pre:C-->E:program-->dropouts lower
(A) There was a recession that caused a high level
of unemployment in the city. 正解,他因削弱
(B) The morale of students who dropped out of
high school had been low even before they
reached high school. 不能削弱,正巧说明这起作用了
(C) As in the preceding year, more high school
students remained in school than dropped
out. 但是不能削弱减少的事实,与是不是那个cause也无关
(D) High schools in the city established placement
offices to assist their graduates in obtaining
employment. 不能削弱,没说没有帮助就找不到工作,那我还是可以辍学提前去找工作。且这项帮助也许之前就是有的。
(E) The antidropout program was primarily aimed
at improving students’ morale in those high
school with the highest dropout rates. 无关,不能削弱。

2. 32s D
3. 1:15 D
4. 34s C 这道题后来看选项思考了比较久,似乎之前理解题意理解错了 remarkable consitency over the past three years 是指两个部门各自的销量之类的连续三年保持不变,而不是两个部门的三年相等。 for the balance应该理解为剩下部分就是chemicals division account的了。。。 这么理解的情况下选择了C
5. 1:11s C

作者: spencerchan    时间: 2015-8-19 12:19
42’’
B: High school students who feel that they are not succeeding in high school often drop out before graduating and go to work
P: These year’s dropout rate is lower than last year’s
C: The program works
Prep: There are other causes to reduce dropout rate, the program is not the key cause
A: CORRECT
B: Irrelevant
C: Not strengthen, not weaken
D: Irrelevant
E: Irrelevant

37’’
B: The climate condition is same in country X and country Y
P: Country X’ production decline, country Y’s production increase
C: The reason is that the new government come to power
Prep: There are other causes => The crop is different?
A: Irrelevant
B: Irrelevant
C: Irrelevant
D: CORRECT
E: Irrelevant

54’’
B: Scientists entry their field with the goal of 1) doing important new research 2) accept as their colleagues those with same motivation
P: Those scientists expound science to general audiences
C: Those scientists are not a true colleague
Prep: They do not have the same motivation
A: Irrelevant
B: Irrelevant, do not mention “envy”
C: Does not means that the popularizer did not do any important researches
D: CORRECT
E: Irrelevant

30’’
B: The two divisions performed remarkable consistency over the past three years
P: P division accounted for 20% sales and 40% profit and the chemical division do the balance
Prep: C: ??
A: Not mention
B: It cannot be inferred
C: CORRECT
D: It cannot be inferred
E: Wrong

1’13’’
P: Only bicycle racers interested in bicycle innovation
C: The innovation based on the race standards
Prep: ??
A: Irrelevant
B: Irrelevant
C: CORRECT
D: Irrelevant
E: Irrelevant

三道题目以前做过,第一次全对 哈哈哈
作者: MIA926    时间: 2015-9-3 19:24
20150903 CR小分队-13
【精练】
2. High school students who feel that they are not
succeeding in high school often drop out before
graduating and go to work. Last year, however, the
city’s high school dropout rate was significantly
lower than the previous year’s rate. This is
encouraging evidence that the program instituted
two years ago to improve the morale of high school
students has begun to take effect to reduce dropouts.
Which one of the following, if true about the last
year, most seriously weakens the argument?
----------Cause and Effect Problems NO.2
计时:28.47s
逻辑链:去年退学率降低,就说是学校展开的打起的program起了作用,求削弱
推测:还要和去年的情况有关,很可能是因为比如来的人少了 或者是家长也进行了一些心理辅导,反正就不关学校的事就是了
(A) There was a recession that caused a high level
of unemployment in the city. 正确 因为找不到工作所以不辍学了
B) The morale of students who dropped out of
high school had been low even before they
reached high school. 无关
(C) As in the preceding year, more high school
students remained in school than dropped
out. 无关
(D) High schools in the city established placement
offices to assist their graduates in obtaining
employment. 无关
(E) The antidropout program was primarily aimed
at improving students’ morale in those high
school with the highest dropout rates. 无关

【逻辑链】
49.
An overly centralized economy, not the changes in the climate, is responsible for the poor agricultural production in Country X since its new government came to power.  Neighboring Country Y has experienced the same climatic conditions, but while agricultural production has been falling in Country X, it has been rising in Country Y.
Which of the following, if true, would most weaken the argument above?
逻辑链:X农作物产量下降,Y上升,就说是X的centralized economy的问题,而不是因为气候变化引起的
推测:GAP很多啊,两国对比的时候没有说明其他条件是否一致,很可能种的东西都不一样有什么好比的
(A) Industrial production also is declining in Country X. 加强
(B) Whereas Country Y is landlocked, Country X has a major seaport. 无关
(C) Both Country X and Country Y have been experiencing drought conditions. 加强
(D) The crops that have always been grown in Country X are different from those that have always been grown in Country Y. 正确
(E) Country X's new government instituted a centralized economy with the intention of ensuring an equitable distribution of goods. 无关

50.
Generally scientists enter their field with the goal of doing important new research and accept as their colleagues those with similar motivation.  Therefore, when any scientist wins renown as an expounder of science to general audiences, most other scientists conclude that this popularizer should no longer be regarded as a true colleague.
逻辑链:科学家很傲娇 只把那些以doing important new research为目标的人看做自己的同事 对于有名望的人都是不屑一顾的
推测:不屑一顾是因为觉得他们的目标不同,道不同不相为谋…
The explanation offered above for the low esteem in which scientific popularizers are held by research scientists assumes that
(A) serious scientific research is not a solitary activity, but relies on active cooperation among a group of colleagues无关
(B) research scientists tend not to regard as colleagues those scientists whose renown they envy 无关
(C) a scientist can become a famous popularizer without having completed any important research 无关
(D) research scientists believe that those who are well known as popularizers of science are not motivated to do important new research 正确
(E) no important new research can be accessible to or accurately assessed by those who are not themselves scientists  无关

51.
A company's two divisions performed with remarkable consistency over the past three years:  in each of those years, the pharmaceuticals division has accounted for roughly 20 percent of dollar sales and 40 percent of profits, and the chemicals division for the balance.
逻辑链:P 0.2 sales 0.4 profit 2   C 0.8sales 0.6 profit 3/4
Which of the following can properly be inferred regarding the past three years from the statement above?
(A) Total dollar sales for each of the company's divisions have remained roughly constant. 新信息
(B) The pharmaceuticals division has faced stiffer competition in its markets than has the chemicals division. 新信息
(C) The chemicals division has realized lower profits per dollar of sales than has the pharmaceuticals division. 正确
(D) The product mix offered by each of the company's divisions has remained unchanged. 新信息
(E) Highly profitable products accounted for a higher percentage of the chemicals division's sales than of those of the pharmaceuticals division. 新信息


52.
The technological conservatism of bicycle manufacturers is a reflection of the kinds of demand they are trying to meet.  The only cyclists seriously interested in innovation and willing to pay for it are bicycle racers.  Therefore, innovation in bicycle technology is limited by what authorities will accept as standard for purposes of competition in bicycle races.
逻辑链:自行车生产商很保守,赛车手则是唯一会对创新感兴趣的人,所以政府会显著自行车行业的 创新,因为觉得这是为了竞争用的
Which of the following is an assumption made in drawing the conclusion above?
(A) The market for cheap, traditional bicycles cannot expand unless the market for high-performance competition bicycles expands. 无关
(B) High-performance bicycles are likely to be improved more as a result of technological innovations developed in small workshops than as a result of technological innovations developed in major manufacturing concerns. 无关
(C) Bicycle racers do not generate a strong demand for innovations that fall outside what is officially recognized as standard for purposes of competition. 正确
(D) The technological conservatism of bicycle manufacturers results primarily from their desire to manufacture a product that can be sold without being altered to suit different national markets.无关
(E) The authorities who set standards for high-performance bicycle racing do not keep informed about innovative bicycle design. 无关

作者: fesche    时间: 2016-5-12 02:08
high school students drop out if they do not perform good
last year drop out lower than previous year
C: strategy put 2 years ago on moral is effective to reduce drop out
weaken
lower in drop out is not because of the program
他因,可能是因为经济不好找不到工作
A
(A) There was a recession that caused a high level of unemployment in the city.
(B) The morale of students who dropped out of high school had been low even before they reached high school.  和现在无关
(C) As in the preceding year, more high school students remained in school than dropped out. 没有解释为什么drop out少了
(D) High schools in the city established placement offices to assist their graduates in obtaining employment. 和drop out during study无关
(E) The anti dropout program was primarily aimed at improving students’ morale in those high school with the highest dropout rates.  和现象无关

falling in agriculture in X is not because of climate but overly centralized economic
BG: climate is the same in X and in Y
P: overly centralized economic
C: poor production in agriculture in X
weaken
因:集中经济
果:农业产出少
Y也是集体经济
D
(A) Industrial production also is declining in Country X. 和工业无关
(B) Whereas Country Y is landlocked, Country X has a major seaport. 和港口无关
(C) Both Country X and Country Y have been experiencing drought conditions. 不能解释X和Y的农业产出的区别
(D) The crops that have always been grown in Country X are different from those that have always been grown in Country Y.
(E) Country X's new government instituted a centralized economy with the intention of ensuring an equitable distribution of goods 和产出无关

scientists enter a new research team tend to do the same as other colleagues
when one gets popular, the other colleagues do not regard him as a true colleague
一个科学家成名,其他科学家觉得这种受欢迎不能成为一个真正的coworker
前提:要成名应该大家一齐成名
A-D
(A) serious scientific research is not a solitary activity, but relies on active cooperation among a group of colleagues
(B) research scientists tend not to regard as colleagues those scientists whose renown they envy
(C) a scientist can become a famous popularizer without having completed any important research
(D) research scientists believe that those who are well known as popularizers of science are not motivated to do important new research
(E) no important new research can be accessible to or accurately assessed by those who are not themselves scientists

P division 20% sales, 40% profit
C division 80% sales, 60% profit
P比C盈利高
C
A) Total dollar sales for each of the company's divisions have remained roughly constant. 不知道total几多
(B) The pharmaceuticals division has faced stiffer competition in its markets than has the chemicals division. 不知道竞争
(C) The chemicals division has realized lower profits per dollar of sales than has the pharmaceuticals division.
(D) The product mix offered by each of the company's divisions has remained unchanged. 不知道mix 的
(E) Highly profitable products accounted for a higher percentage of the chemicals division's sales than of those of the pharmaceuticals division. 不知道percentage

manufacture decision is reflection on cyclist
manufacture make innovation base on the demand or requirement of authorities
assumption
官方影响新单车的技术
C
A) The market for cheap, traditional bicycles cannot expand unless the market for high-performance competition bicycles expands.
(B) High-performance bicycles are likely to be improved more as a result of technological innovations developed in small workshops than as a result of technological innovations developed in major manufacturing concerns.
(C) Bicycle racers do not generate a strong demand for innovations that fall outside what is officially recognized as standard for purposes of competition.
(D) The technological conservatism of bicycle manufacturers results primarily from their desire to manufacture a product that can be sold without being altered to suit different national markets.
(E) The authorities who set standards for high-performance bicycle racing do not keep informed about innovative bicycle design.

作者: 宋痂岛__    时间: 2017-12-15 16:56
ADDCC
作者: May97    时间: 2018-1-27 13:21
T:27s
BG:High school students who feel that they are not succeeding in high school often drop out before graduating and go to work.
P:Last year, however, the city’s high school dropout rate was significantly lower than the previous year’s rate. C:This is encouraging evidence that the program instituted two years ago to improve the morale of high school students has begun to take effect to reduce dropouts.
Weaken:
(A) There was a recession that caused a high level of unemployment in the city.--->irreverent--->correct 另一个原因
(B) The morale of students who dropped out of high school had been low even before they reached high school.--->irrevrent
(C) As in the preceding year, more high school students remained in school than dropped out.--->support
(D) High schools in the city established placement offices to assist their graduates in obtaining employment.--->correct. 另一个原因这里只是说明dropout rate会提高,irreverent
(E) The antidropout program was primarily aimed at improving students’ morale in those high school with the highest dropout rates.--->support

T: 26s
C: An overly centralized economy, not the changes in the climate, is responsible for the poor agricultural production in Country X
P: since its new government came to power.  
P: Neighboring Country Y has experienced the same climatic conditions, but while agricultural production has been falling in Country X, it has been rising in Country Y.
Weaken: 1) X Y种的植物不同,受到天气影响不同; 2) the new government 并没有centralized economy
(A) Industrial production also is declining in Country X.--->不一定
(B) Whereas Country Y is landlocked, Country X has a major seaport.--->???BD选项比较纠结。。
(C) Both Country X and Country Y have been experiencing drought conditions.--->support
(D) The crops that have always been grown in Country X are different from those that have always been grown in Country Y.--->correct
(E) Country X's new government instituted a centralized economy with the intention of ensuring an equitable distribution of goods.--->support

T: 26s
P: Generally scientists enter their field with the goal of doing important new research and accept as their colleagues those with similar motivation.  
C: Therefore, when any scientist wins renown as an expounder of science to general audiences, most other scientists conclude that this popularizer should no longer be regarded as a true colleague.
FA: research scientist的目的改变了
(A) serious scientific research is not a solitary activity, but relies on active cooperation among a group of colleagues--->irreverent
(B) research scientists tend not to regard as colleagues those scientists whose renown they envy--->irreverent
(C) a scientist can become a famous popularizer without having completed any important research--->irreverent
(D) research scientists believe that those who are well known as popularizers of science are not motivated to do important new research--->correct
(E) no important new research can be accessible to or accurately assessed by those who are not themselves scientists --->irreverent

T: 31s
A company's two divisions performed with remarkable consistency over the past three years:  in each of those years,
the pharmaceuticals division has accounted for roughly 20 percent of dollar sales and 40 percent of profits,
and the chemicals division for the balance.
Infer:
(A) Total dollar sales for each of the company's divisions have remained roughly constant.--->no
(B) The pharmaceuticals division has faced stiffer competition in its markets than has the chemicals division.--->no
(C) The chemicals division has realized lower profits per dollar of sales than has the pharmaceuticals division.--->correct
(D) The product mix offered by each of the company's divisions has remained unchanged.--->no
(E) Highly profitable products accounted for a higher percentage of the chemicals division's sales than of those of the pharmaceuticals division.--->no

T: 33s
P: The technological conservatism of bicycle manufacturers is a reflection of the kinds of demand they are trying to meet.  
P: The only cyclists seriously interested in innovation and willing to pay for it are bicycle racers.  
C: Therefore, innovation in bicycle technology is limited by what authorities will accept as standard for purposes of competition in bicycle races.
FA:
(A) The market for cheap, traditional bicycles cannot expand unless the market for high-performance competition bicycles expands.--->irreverent
(B) High-performance bicycles are likely to be improved more as a result of technological innovations developed in small workshops than as a result of technological innovations developed in major manufacturing concerns.--->irreverent
(C) Bicycle racers do not generate a strong demand for innovations that fall outside what is officially recognized as standard for purposes of competition.--->correct
(D) The technological conservatism of bicycle manufacturers results primarily from their desire to manufacture a product that can be sold without being altered to suit different national markets.--->irreverent
(E) The authorities who set standards for high-performance bicycle racing do not keep informed about innovative bicycle design.--->irreverent
作者: 顿东w    时间: 2018-9-21 08:40
感谢分享!               
作者: 顿东w    时间: 2018-9-21 09:04
2.
B:在校感觉不成功的高中生经常退学。
P:两年前提高学生士气的一个项目起了作用导致
C:去年退学率降低。
(A) There was a recession that caused a high level of unemployment in the city.
The answer attacks the conclusion by introducing an alternate cause: it was not the morale program that led to a decrease in high dropouts, but rather the fact that no jobs were available for individuals contemplating dropping out of high school. The job availability factor is important because the first sentence of the stimulus indicates that high school students who drop out go to work. Thus, if a recession led to a high level of unemployment, this could cause high school students to rethink dropping out and stay in school.
(D) High schools in the city established placement offices to assist their graduates in obtaining employment.
帮助毕业生找到工作,但是没有毕业的高一高二的得不到学校的帮助还是会选择退学的(我开始想的就是既然学校帮助学生们找工作,那么大家肯定都会待到毕业等到学校的帮助这样就不会退学了 但是这个选项针对的仅仅是graduates,没有管其他年级的学生)

49.
XY都经历同样的天气气候,但是X的产量下降,Y的产量却上升。
C:新政权上台之后,过度中央集权的经济应该为X的poor农业生产负责。
maybe:两国种的农作物不同;集权经济是有好处的
D

50.
P:科学家带着做重要研究的心进入这个领域并且容纳接受有着相同目标的同事。
C:一旦有人成为了大众的解释者,其他的科学家no longer regard him as a true colleague
maybe:一旦受了欢迎,就没有相同的目标了
(A) serious scientific research is not a solitary activity, but relies on active cooperation among a group of colleagues
(B) research scientists tend not to regard as colleagues those scientists whose renown they envy
(C) a scientist can become a famous popularizer without having completed any important research
(D) research scientists believe that those who are well known as popularizers of science are not motivated to do important new research
(E) no important new research can be accessible to or accurately assessed by those who are not themselves scientists

51.
制药部门卖了20%的sales,盈利40%;化学部门卖了80%的sales,盈利60%
infer:制药部门的利润效率比化学的高
(A) Total dollar sales for each of the company's divisions have remained roughly constant.
(B) The pharmaceuticals division has faced stiffer competition in its markets than has the chemicals division.
(C) The chemicals division has realized lower profits per dollar of sales than has the pharmaceuticals division.
(D) The product mix offered by each of the company's divisions has remained unchanged.
(E) Highly profitable products accounted for a higher percentage of the chemicals division's sales than of those of the pharmaceuticals division.

52.
自行车产商的技术保守主义是他们需要去迎合的demand的一种reflection
对创新感兴趣并且愿意花钱的是那些自行车手。
C:自行车技术的创新局限于权威在自行车比赛里会将什么作为标准。
mayeb:自行车手就是那些权威们
(A) The market for cheap, traditional bicycles cannot expand unless the market for high-performance competition bicycles expands.
(B) High-performance bicycles are likely to be improved more as a result of technological innovations developed in small workshops than as a result of technological innovations developed in major manufacturing concerns.
(C) Bicycle racers do not generate a strong demand for innovations that fall outside what is officially recognized as standard for purposes of competition.
(D) The technological conservatism of bicycle manufacturers results primarily from their desire to manufacture a product that can be sold without being altered to suit different national markets.
(E) The authorities who set standards for high-performance bicycle racing do not keep informed about innovative bicycle design.
作者: echo-LUO    时间: 2018-9-22 18:30
精炼:
高中生如果不爱上学了就辍学工作——>今年辍学率降低——>肯定是前两年加强高中士气起效果
weaken——>其他因素导致辍学率降低;证明morale不会降低辍学率
(A) There was a recession that caused a high level of unemployment in the city.——>不好就业,所以找不到工作,所以辍学率降低
(B) The morale of students who dropped out of high school had been low even before they reached high school.——>应该说的是为什么会有人数变化,无关
(C) As in the preceding year, more high school students remained in school than dropped out.——>无关
(D) High schools in the city established placement offices to assist their graduates in obtaining employment.——>无关
(E) The antidropout program was primarily aimed at improving students’ morale in those high school with the highest dropout rates.——>无关



逻辑链:
49.

X农业收成不好是因为新政府过于中心化的经济,而不是气候原因——>隔壁Y国,同样气候收成就挺好
weaken——>X和Y种的不是一个东西;Y的地形etc
(A) Industrial production also is declining in Country X.——>无关
(B) Whereas Country Y is landlocked, Country X has a major seaport.——>无关
(C) Both Country X and Country Y have been experiencing drought conditions.——>应该找不同,加强
(D) The crops that have always been grown in Country X are different from those that have always been grown in Country Y.——>农作物不一样,削弱
(E) Country X's new government instituted a centralized economy with the intention of ensuring an equitable distribution of goods.——>无关



50.
科学界都悦纳投身科研,有共同目的的科学家——>如果一个人转向大众传播,博取名声,会被厌弃
assumption: an expounder of science to general audiences就不是重要科研研究,对科学没有追求
(A) serious scientific research is not a solitary activity, but relies on active cooperation among a group of colleagues
(B) research scientists tend not to regard as colleagues those scientists whose renown they envy
(C) a scientist can become a famous popularizer without having completed any important research——>推广者没有重要研究,所以不招人待见
漏洞:稳重针对的是所有做科学传播获得名誉的科学家,但这里只说到了其中一部分
(D) research scientists believe that those who are well known as popularizers of science are not motivated to do important new research——>受大众欢迎的被认为没有动力去作新研究
(E) no important new research can be accessible to or accurately assessed by those who are not themselves scientists


51.
p部门20%销量,40%利润;c部门80%销量,60%利润
(A) Total dollar sales for each of the company's divisions have remained roughly constant.
(B) The pharmaceuticals division has faced stiffer competition in its markets than has the chemicals division.
(C) The chemicals division has realized lower profits per dollar of sales than has the pharmaceuticals division.——>单位利润,c<p
(D) The product mix offered by each of the company's divisions has remained unchanged.
(E) Highly profitable products accounted for a higher percentage of the chemicals division's sales than of those of the pharmaceuticals division.


















































作者: 云栈    时间: 2019-11-2 05:29
1.        24s
P:感觉自己没有在学校不会成功的高中生,往往在毕业前辍学去上班。
P:但是,去年,该市的高中辍学率明显低于上一年。
C: 表明两年前制定的提高高中生士气的计划已开始生效以减少辍学率。
推测:原题的因果关系为学校提高学生的士气导致辍学率下降。证明因果关系反了或者还有其他原因导致了这个结果。
选项分析:A
A: 失业率高说明学生不好找工作所以他们不会辍学
B:学生辍学前后的士气和他们是否会辍学没关系
C: 在校学生比辍学的多和原题没关系
D: 帮助已经毕业的学生找工作和辍学学生无关
E: 原题说的是城市的overall辍学率,不是某个学校的辍学率
2.        35s
P: 邻国Y经历了相同的气候条件,但是X国的农业生产一直在下降,而Y国却在增长。
C: 自从X国新政府上台以来,过度集中的经济而非气候的变化是X国农业生产不佳的原因。
推测:确实是天气影响了农作物产量;或者不是经济而是其他原因导致;比如两个城市种的东西不一样
选项分析:D

3.        43s
P: 通常,科学家进入其领域的目的是进行重要的新研究,并接受具有类似动机的科学家作为他们的同事。
P: 因此,当任何一位科学家通过向大众普及科学而获得声誉时,大多数其他科学家得出的结论是,不应将此普及者视为真正的同事。
推测:这些普及科学的人并不是跟那些科学家一起做调研的人;那些普及科学的人对research没有motivation
选项分析:D
【这道做错了,选了C,不仔细看题是硬伤】

4.        49s
P:一个公司的两个部门在过去三年中表现出了惊人的一致性:在那些年中,制药部门每年约占销售额的20%,利润占40%。
推测:化学division承包80%的sales和60%的profit;药理的利润高
选项分析:C

5.        48s
P: 自行车制造商的技术保守性反映了他们试图满足的各种需求。
P:唯一对创新有浓厚兴趣并愿意为此付出代价的自行车手是自行车赛车手。
C:因此,以自行车比赛为目的的自行车技术的创新受到当局所接受的标准的限制。
推测:科技改变带来的利润不高,但是对赛车手的提升大;如果制造商不愿意提高,赛车手也无法提高自行车科技
选项分析:C
【逻辑点没太懂,排除法得出的答案】

作者: Lincy123    时间: 2020-5-7 22:11
P:去年辍学率降低
C:两年前的项目有效果
A)        一个recession导致高的失业率。Correct
B)        辍学学生的morale在高中之前就很低
C)        近年来更多高中学生待在学校而不是辍学。无关
D)        高中成立了办公室帮助学生就业。无关
E)        这个项目最初着重在辍学了最高的高中,无关

P:隔壁Y遭到同样的气候条件,但农业生产上涨
C:是集中经济导致X不好的农业生产
A)        X的工业生产也降低。无关
B)        尽管Y landlocked, X有主要的港口。无关
C)        X和Y都经理了干旱。无关
D)        X和Y的农作物不同,correct
E)        X的新政府发布了干涉商品分配的中央经济。无关

P:科学家接受类似动机的同事
C:受大众欢迎的科学家不再是真正的同事
Gap:受欢迎和doing important new research的关联
A)        科学研究需要合作。无关
B)        受到嫉妒的科学家不会被当作同事。
C)        没有完成任何重要研究的科学家也可以非常受欢迎。取非 – 有完成重要研究的科学家也可受欢迎。跟new无关
D)        有名的科学家没有动力做新研究。取非-有名的科学家有动力做新研究,weaken,correct
E)        没有新研究可以精确地被哪些不是科学家的人取得。无关

每一年,药物部门占了20%的销售额,40%的利润,化学部门for the balance(80%销售额,60%利润)
Must be true:化学部门成本高
A)        总销售额不变。无法推出
B)        药物部门竞争更激烈。无法推出
C)        化学部门的每一销售额利润比药物部门低。Correct
D)        综合产品每年不变。无法推出
E)        高利润产品在化学部门销售中占比高,无法推出

P:只有自行车比赛选手对自行车创新感兴趣而且愿意买单
C:自行车创新科技受到官方自行车比赛的标准限制
Gap:自行车比赛选手只对比赛的标准方面创新感兴趣
A)        除非高性能比赛自行车市场扩增,便宜传统的自行车市场不会扩增。无关
B)        自行车科技创新通常发生在小作坊,而不是大型制造工厂,无关
C)        自行车比赛选手不会对比赛限制之外的创新感兴趣,correct
D)        科技创新限制来源于他们想制造不会为了顺应不同国家市场而改造的产品。无关
E)        指定标准的权威不会被告知创新的自行车设计。无关

第一次做对




欢迎光临 ChaseDream (https://forum.chasedream.com/) Powered by Discuz! X3.3